CPRH

Ace your homework & exams now with Quizwiz!

A 43-year-old Caucasian woman presents to the ER with RUQ pain, vomiting and fever for the last two hours. She has a BMI of 34. Which is the best screening test to help diagnose this patient? A. CXR- AP view B. AXR C. Abdominal US D. Abdominal CT scan E. CXR- PA view

C. Abdominal US

A 9-year-old boy presents to the pediatrician with peri-umbilical pain, fever and vomiting for the last three hours. Which of the following is the best initial test for evaluation? A. Abdominal CT scan. B. Abdominal US C. Abdominal MRI D. CXR E. Chest CT Scan

B. Abdominal US

Based on the physical findings, which of the following valvular heart diseases is the most likely diagnosis in this patient? A. Mitral stenosis B. Aortic stenosis C. Mitral regurgitation D. Aortic regurgitation E. Pulmonary stenosis F. Pulmonary regurgitation G. Tricuspid stenosis H. Tricuspid regurgitation

A. Mitral stenosis

A 29-year-old man comes to the physician because of a 4-week history of episodes of light headedness and palpitations. He says that each episode lasts for a few seconds. His pulse at rest is 74/min and regular. An ECG in lead II taken during an episode is shown below. Which of the following is the most likely cause of this patient's palpitations? A. 1st degree heart block B. 2nd degree heart block-type1 C. Long QT syndrome D. WPW syndrome E. 2nd degree heart block-type2

C. Long QT syndrome

A 42-year-old man comes to the physician because of a 2-week history of hemoptysis, hoarseness and shortness of breath. He says that the shortness of breath worsens in supine position. There is no history of chronic cough, fever or chest pain. He had rheumatic fever when he was an adolescent. Which of the following differentials are most likely to be considered? (you may choose more than 1) A. Valvular heart disease B. Anemia C. Lung cancer D. Congenital heart disease E. Coronary artery disease F. Pulmonary tuberculosis G. Arteriovenous fistula

A, C, D

A 25-year-old man comes to the physician because of a 1-week history of increasing malaise, dark colored urine and decreasing urine output. His temperature is 98.4°F(37°C) and B.P is 170/116mm Hg. Physical examination shows ankle edema. Laboratory tests and renal biopsy are shown +2 protein; +4 blood >40 RBCs/HPF 10 WBCs/HPF 5-10 RBC casts/LPF CR: 3.2 mg/dL Which of the following is considered for a differential diagnosis? A. Anti GBM disease B. Lupus nephritis C. Minimal change disease D. Membranous nephropathy E. Microscopic polyangitis

A. Anti GBM disease B. Lupus nephritis E. Microscopic polyangitis

A 25-year-old female comes to the physician because of a 3-week history of persistent blood in urine. Her temperature is 98.4°F(37°C) and blood pressure is 170/126mm Hg. Physical examination shows 2+ ankle edema. Laboratory studies show an elevated serum creatinine and decreased GFR. Urinalysis shows 2+ protein, 3+ RBCs and RBC casts. Renal biopsy shows large, hypercellular glomeruli with mesangial proliferation, along with thickening and splitting of the basement membrane. Which of the following tests should be ordered to further evaluate this patient? (Multiple correct options) A. Antinuclear antibodies B. Hepatitis B and C serology C. Blood cultures bacteremia D. Serum electrophoresis E. Serum C3, C4 levels F. C3 nephritic factor (C3NeF) antibodies

A. Antinuclear antibodies B. Hepatitis B and C serology C. Blood cultures bacteremia D. Serum electrophoresis E. Serum C3, C4 levels F. C3 nephritic factor (C3NeF) antibodies

2. A 75-year-old man comes to the physician for a yearly checkup. Measurement of his blood pressure shows a systolic pressure of 150 mm Hg and a diastolic pressure of 78 mmHg. Physical examination shows no abnormalities. A decrease in which of the following cardiovascular factors is the most likely cause of his high pulse pressure? A. Arterial compliance B. Cardiac output C. Myocardial contractility D. Stroke volume E. Total peripheral resistance

A. Arterial compliance

In the above question, -Antinuclear antibody (ANA) is negative-Antibodies against hepatitis B core (HBc) antigen, HCV is negative.-Blood culture is sterile-C3 levels are low-Immunofluorescence shows positive staining for complement C3 and absent staining for Ig. -Electron microscopy shows dense, ribbon-like appearance of subendothelial and intramembranous and narrowing of the capillary lumen.What is the most likely diagnosis? A. Dense deposit disease B. C3 Glomerulonephritis C. Immune complex mediated RPGN D. Lupus nephritis E. Anti GBM disease

A. Dense deposit disease

A 3mo infant with Tetralogy of Fallot is emergently transferred from cardiology clinic to the PICU with profound cyanosis (SaO2=25%). HR 188(high), Bp 70/55(borderline), Morphine, IVF, NaHCO3 (Sodium Bicarbonate) have been given and the baby is now intubated without improvement. The following infusion should be started. A. Esmolol B. Epinephrine C. NaHCO3 D. Vecuronium

A. Esmolol

Clicker question: Which of the following is/are possible blood profiles in a patient with CKD-related abnormalities in calcium and phosphate metabolism? A. Hyperphosphatemia B. Hypocalcemia C. Hypercalcemia D. Hyperparathyroidism

A. Hyperphosphatemia B. Hypocalcemia C. Hypercalcemia D. Hyperparathyroidism

A balloon-tipped catheter is placed into a small branch of the pulmonary artery as shown. The lumen of the catheter opens to the balloon. The pressure measured from the tip of the catheter with the volume deflated is 25/8 mmHg. When the balloon is inflated the pressure is 12 mmHg and non-pulsatile. Which of the following pressures is being approximated When the balloon is inflated? Session ID: sidowu A. Left atrial pressure B. End diastolic pressure in LV C. Peak systolic pressure in LV D. Pulmonary artery pressure E. Right atrial pressure

A. Left atrial pressure

10. A 60-year-old woman comes to the physician because of a 4-month history of fatigue and paroxysmal nocturnal dyspnea. Her pulse is 90/min, blood pressure is 166/56 mmHg, and respirations are 15/min. Examination of her peripheral arterial pulses shows a rapidly rising pulse with a collapse. Physical examination shows an apical impulse in the sixth intercostal space along the anterior axillary line, an S3 gallop and a high-pitched, blowing, early diastolic decrescendo murmur heard best along the left lower sternal margin. Which of the following pathological changes is most likely in this patient's heart? A. Left ventricular hypertrophy of eccentric type B. Left ventricular hypertrophy of concentric type C. Right ventricular hypertrophy of eccentric type D. Right ventricular hypertrophy of concentric type E. Diastolic dysfunction of the left ventricle

A. Left ventricular hypertrophy of eccentric type (aortic regurgitation)

In the above question : Three days later, the patient is brought to the physician because of worsening of her presenting symptoms. Her temperature is 97.8 °F (36.5°C), pulse is 100/min and blood pressure is 170/110 mm Hg. Light microscopy of the renal biopsy is shown in figure. Which of the following cells would most likely be present in this lesion? A. Macrophages B. Megakaryocytes C. Astrocytes D. Neutrophils E. Microcytic RBCs

A. Macrophages

A 75-year-old man with a past medical history of hypertension, diabetes mellitus, COPD, and gout presents for a health maintenance visit. He has been stable over the last year without hospitalizations. His most recent pulmonary function tests show an FEV1 of 45%. Vitals signs: blood pressure 142/68 mmHg, pulse 82/min, respiratory rate 14/min, and oxygen saturation 92% on room air. On physical examination, the patient breathes slowly with prolonged expiration. Which of the following is a potential complication of this patient's underlying pulmonary disease? A. Polycythemia B. Hypernatremia C. Anemia D. Neuropathy E. Arthritis

A. Polycythemia

A 12-year-old boy comes the physician because of 2-weeks history of dark brown colored urine and pain in lower abdomen. He also has a similar history of 3 episodes of hematuria in last 2 years. There is also a family history of renal disease. His temperature is 98.4°F(37°C), pulse is 75/min, respirations are 15/min and B.P is 150/95mm Hg. Physical examination shows mild sensorineural deafness bilaterally. Urinalysis shows 2+ proteinuria and 3+ hematuria. Laboratory tests show creatinine 3.1 mg/dL (0.6-1.2 mg/dL) and BUN 48 mg/dL (7-18 mg/dL). Which of the following glomerular basement membrane (GBM) changes will be most likely seen on electron microscopy? A. Segmentally thinned and thickened areas with splitting/lamination B. Wrinkling and disruption C. Ribbon-like electron-dense deposits D. Thickening and double contour E. Diffuse thickening

A. Segmentally thinned and thickened areas with splitting/lamination

Based on the clinical features and ECG, which of the following tachyarrhythmias is/are most likely differentials in this patient? (you may select more than 1 option) A. Sinus tachycardia B. Atrial flutter C. Multifocal atrial tachycardia D. Supraventricular tachycardia E. AV nodal reentry tachycardia F. Sustained Ventricular tachycardia G. AV reentry tachycardia H. Atrial tachycardia (focal atrial tachycardia) I. Narrow complex tachycardia

A. Sinus tachycardia D. Supraventricular tachycardia H. Atrial tachycardia (focal atrial tachycardia) I. Narrow complex tachycardia

A 46-year-old woman comes to the emergency department because of a 2-hour history of sudden onset of palpitations, lightheadedness, and shortness of breath. She does not have any medical conditions. These symptoms began approximately 2 hours ago. PE: BP 95/70 mm Hg; Heart Rate - averages 170/min, regularRest of her physical examination is unremarkable. ECG in lead II: retrograde (negative) P waves; short R-P intervals and narrow QRS complexes.Which of the following is the most likely diagnosis? A. Supraventricular Tachycardia B. Sinus tachycardia C. Ventricular tachycardia D. Acute myocardial infarction E. Pulmonary embolism

A. Supraventricular Tachycardia

3. A 51-year-old man comes to the physician because of a 2-month history of dyspnea on exertion and chest discomfort. He has Marfan syndrome. His pulse is 96/min and is of collapsing type, and blood pressure is 164/64 mmHg. Physical examination shows a laterally shifted and hyperdynamic apical impulse, high-pitched diastolic murmur in the left sternal border and head bobbing. An x-ray of his chest shows enlargement of left ventricle. Which of the following hemodynamic alterations on the left ventricular function is the most likely cause of his symptoms? A. Volume overload B. Pressure overload C. Restricted filling D. Loss of myocardial contractility E. Loss of myocardial tissue

A. Volume overload

A 41-year-old man is referred to the cardiologist because of a heart murmur. He had applied for a job that was denied because of the murmur. He is asymptomatic and physically active. There is no history of chest pain, dyspnea, or spells of syncope. There is no family history of heart diseases. He has never had high blood pressure or diabetes, doesn't smoke, and takes no medications. He had infective endocarditis 5 years ago. Which of the following differentials are most likely to be considered? (You may choose more than 1) A. Valvular heart disease B. Anemia C. Thyrotoxicosis D. Congenital heart disease E. Coronary artery disease F. Cor pulmonale G. Arteriovenous fistula

A., B., D., G.

A 72-year-old man with Marfan syndrome comes for a follow up examination. He says that his sleep is disturbed by worsening shortness of breath for the past 2 weeks. His pulse is 92/min and blood pressure is 168/68 mm Hg. Physical examination shows downward and outward shifted apical impulse and a harsh early diastolic murmur in Erb's area. In addition, multiple abnormal sounds are heard in other auscultatory areas. Which of the following additional auscultatory findings are most likely in this patient? (choose more than one) A. Holosystolic murmur B. Systolic flow murmur C. Late systolic murmur D. Aortic ejection click E. Mid-diastolic murmur F. Continuous murmur G. Audible S4

B and E (severe aortic regurgitation)

A 43-year-old man comes to the physician because of an 8-week history of pruritis, lethargy and swelling in both legs. He has a 10-year history of hypertension and type 2 Diabetes Mellitus. His blood pressure is 180/110mm Hg, pulse is 82/min and respirations are 12/min. Laboratory studies show:Blood glucose: 110 mg/dL (N= 70-100 mg/dL)Serum sodium: 133 mEq/L (N= 136-146 mEq/L)Serum potassium: 6.3 mEq/L (N=3.5-5.0 mEq/L )BUN 170mg/dl (N=7-22 mg/dl)Serum Creatinine 10.0 mg/dl (N= 0.6-1.2mg/dl)24-hour urine: Volume 1440ml/day, protein 600 mg/dl and creatinine 200mg/dl.What is the estimated GFR ? A. 2 ml/min B. 20ml/min C. 28.8ml/min D. 288ml/min E. 120ml/min

B. 20ml/min

8. A 52-year-old woman comes to the physician because of a 4-month history of progressive dyspnea on exertion and an uncomfortable awareness of pulsations in the neck and chest. Physical examination shows water-hammer type peripheral pulses, head bobbing and hyperdynamic, laterally displaced apical impulse. Cardiac auscultation shows a high pitched, blowing, decrescendo type diastolic murmur in the left lower sternal border. Which of the following findings is most likely from this patient's arterial blood pressure recording? A. Elevated diastolic pressure B. A wide pulse pressure C. Low systolic pressure D. A significant difference in systolic pressures between expiration and inspiration E. A significant difference between upper arm and lower arm BP

B. A wide pulse pressure

An alert, 6 month old male has a history of vomiting and diarrhea. He appears pale and has an RR of 45 breaths per minute, HR of 180 beats per minute, and a systolic blood pressure of 85 mm Hg. His extremities are cool and mottled with a capillary refill time of 4 seconds. What would best describe his circulatory status? A. Normal circulatory status B. Early (compensated) shock caused by hypovolemia C. Early (compensated)shock caused by supraventricular tachycardia D. Late (decompensated) shock caused by hypovolemia E. Late (decompensated) shock caused by supraventricular tachycardia

B. Early (compensated) shock caused by hypovolemia

6. A 74-year-old man comes to the physician because of a 4-month history of chest heaviness and shortness of breath. He had 2 episodes of syncope in the last 2 weeks. Pressure profile during cardiac catheterization is shown below. Which of the following physical findings is most likely in this patient? A. Late systolic murmur B. Ejection click in aortic area C. Opening snap in mitral area D. Middiastolic murmur E. Collapsing arterial pulse

B. Ejection click in aortic area

Clicker question: Which of the following is the most appropriate assessment of this patient's kidney status? A. He has acute kidney injury B. He has chronic kidney disease C. He has normal, healthy kidneys

B. He has chronic kidney disease

If this patient were a child and had arthritis, vasculitis and purpura (ecchymosis and petechie), what would be the likely diagnosis? A. Alport syndrome B. Henoch-Schönlein purpura C. Idiopathic thrombocytopenic purpura D. Thrombotic thrombocytopenic purpura E. Wiskott-Aldrich syndrome

B. Henoch-Schönlein purpura

Following an MVA, a 30 yo woman unconscious in the ED, blood pressure is 64/40 mm Hg, heart rate is 150 bpm. intubated and hand-ventilated. What pathophysiology will result in death the quickest A. Acute Respiratory failure B. Hypovolemic Shock C. Chronic renal disease D. Acute Arrythmia E. Stroke

B. Hypovolemic Shock d/t intrabdominal hemorrhage

A 48-year-old man comes to the physician for a follow up examination. He is obese and chronic smoker. He has a 5-year history of hypertension and angina pectoris. Physical examination shows no abnormalities. He is advised to undergo an angiogram to determine his cardiac status. The result shows a critical narrowing of the circumflex artery. Which of the following myocardial regions is most likely ischemic in this patient? A. Anterior wall of the left ventricle B. Lateral wall of the left ventricle C. Inferior wall of the left ventricle D. Posterior wall of the left ventricle E. Septal wall

B. Lateral wall of the left ventricle

A 51-year-old man comes to the physician because of a 6-month history of palpitations and chest discomfort. His pulse is 88/min and blood pressure is 168/58 mm Hg. Physical examination shows hyperdynamic, outward shifted apical impulse and an early diastolic murmur heard along the left sternal border. Left-sided pressure recording is done. Which of the following findings is most likely? A. Low peak systolic LV pressure B. Low aortic diastolic pressure C. Elevated left atrial pressure D. Low aortic systolic pressure E. Low pulse pressure F. A wide gap between peak systolic LV pressure and aortic systolic pressure

B. Low aortic diastolic pressure

1. A 65-year-old man comes to the physician because of a 4-week history of palpitations, poor exercise tolerance, generalized weakness and dizziness. He says that he had two episodes of sudden loss of consciousness for a few seconds during this period. His pulse ranges between 75-125/min and is irregularly irregular and blood pressure is 115/75 mm Hg. An ECG in lead II is shown below. Which of the following electrophysiologic abnormalities most likely initiated this patient's condition? A. Multiple impulses generated at multiple sites in the ventricle B. Multiple impulses generated at multiple sites in the atrium C. Rapid impulses generated in one focus in the atrium D. Rapid impulses generated in one focus in the ventricle E. Rapid impulses generated in the AV done F. Rapid impulses generated in the SA node

B. Multiple impulses generated at multiple sites in the atrium

Clicker question: In reviewing the pathophysiology of diabetic kidney disease, which of the following biochemical pathways and underlying reactions are correctly paired? A. Polyol pathway -> conversion of NADP+ to NADPH B. Protein kinase C activation -> upregulation of diacylglycerol C. Hexosamine pathway -> decreased transcription of TNF-α D. Formation of AGEs -> decreased production of methylglyoxal

B. Protein kinase C activation -> upregulation of diacylglycerol

A 50-year-old social worker comes to the physician because of difficulty in breathing and feeling of fatigue since couple of months. It usually aggravates on lying down and often wakes him up at night. He had suffered a cardiac arrest 4 years back, following which angiography was done. His respiration is 28/min, heart rate is 94/min. On examination of pulses, which of the following patterns is likely to be noticed? Session ID: sidowu A. Pulses bisferiens B. Pulsus alternans C. Pulsus paradoxus D. Collapsing pulse

B. Pulsus alternans

A 48-year-old man comes to the physician because of episodes of chest pain. An ECG is recorded during treadmill and ST depression changes are seen in the leads II, III & aVF. Which of the following blood vessels is most likely blocked in him? A. Left coronary artery B. Right coronary artery C. Left circumflex artery D. Left anterior descending (LAD) artery E. Septal branches of LAD artery

B. Right coronary artery

On PE there is noted +2 pitting edema up to the knee.What is the mechanism causing the edema? A. She's been wearing tunicates for fun B. She is retaining sodium C. She drinks too much water D. She has a DVT

B. She is retaining sodium

4. A 55-year-old woman comes to the physician because of a 2-week history of fatigue and light-headedness. She was diagnosed with inferior wall MI 6 months ago. Her pulse is 56/min and is regular in rhythm. Physical examination shows normal JVP but occasional prominent "a" waves and a loud S4. An ECG in lead II is shown below. Which of the following best explains the physical examination finding in this patient? A. Ectopic generation of atrial rhythm B. Simultaneous contraction of both atria and ventricles C. Stronger atrial contraction due to hypertrophy D. Stronger ventricular contraction due to hypertrophy E. Prolonged ventricular filling due to bradycardia

B. Simultaneous contraction of both atria and ventricles

1. A 70-year-old man comes to the emergency department because of a 30-min history of shortness of breath and an episode of syncope. He says he has shortness of breath for the past 8 month. His pulse ranges from 88 to 124/min and is irregularly irregular, and blood pressure is 96/60 mmHg. Physical examination shows a crescendo-decrescendo systolic murmur heard at the upper right sternal border radiating to the carotids and bilateral basal rales. Echocardiography shows left ventricular hypertrophy. An ECG shows absent P waves and irregular R-R intervals. Which of the following hemodynamic changes most likely contributed to this patient's presentation? A. Sudden decrease in the left ventricular contractility B. Sudden decrease in the ventricular preload C. Sudden increase in the left ventricular afterload D. Sudden increase in the left ventricular filling E. Right ventricular hypertrophy

B. Sudden decrease in the ventricular preload

4. A 25-year-old man is brought to the emergency department because of a 4-week history of dyspnea and palpitation on exertion. He is diagnosed with long-QT syndrome 6 months ago. His ECG during palpitation is shown below. Which of the following mechanisms is most likely in the genesis of tachyarrhythmia in this patient? A. Accelerated SA nodal rhythm B. Triggered activity during afterdepolarization C. Accelerated AV nodal rhythm D. Reentrant circuit in the ventricle E. Atrioventricular reentry

B. Triggered activity during afterdepolarization (Torsades de pointes)

A 45 yo male smoker, BMI 41, presents to the office with intermittent headaches especially in the morning, waking at night and snoring. What is the cause for his headaches? A. Brian tumor B. Untreated hypertension C. Loud and obnoxious children D. Acute Renal Failure E. Chronic Renal Failure

B. Untreated hypertension

A 62-year-old diabetic man underwent an abdominal aortic aneurysm repair 2 days ago. He is being treated with gentamicin for a urinary tract infection. His urine output has fallen to 300 mL over 24 hours, and his serum creatinine has risen from 1.1 mg/dL on admission to 1.9 mg/dL. In the CICU fluid management was very conservative to make sure the patients did not get too puffy due to the normal SIRS response with surgery. Which of the following laboratory values would be most consistent with his etiology of his renal insufficiency? A. FENa of 3% B. Urinary sodium level of 10 mEq/L C. Central venous pressure reading of 10 mm Hg D. Gentamicin trough level of 4 µg/mL

B. Urinary sodium level of 10 mEq/L

5. A 65-year-old man is brought to the emergency dept. because of a 1-hour history of crushing chest pain. An ECG recorded on arrival shows ST segment elevation in leads I, aVL and V5-V6. His BP is 96/54 mmHg. He is being monitored in the intensive care unit. A Swan-Ganz catheter is introduced through his neck vein and the pressure recordings are obtained. Which of the following sets of findings is most likely in this patient? A. inc PCWP, dec CVP and SVR B. inc PCWP, CVP and SVR C. dec PCWP, CVP and SVR D. dec PCWP, inc CVP and SVR E. inc PCWP, dec CVP and inc SVR

B. inc PCWP, CVP and SVR (cardiogenic shock because your LV muscle is weak so you cant pump out as much so dec CO)

8. A 75-year-old woman comes to the emergency dept. because of a 1-hour history of crushing substernal chest pain. An ECG taken on arrival in V5 & V6 is shown. Her blood pressure is 112/62 mmHg but is stable and, she is alert. Coronary angiogram shows functional occlusion of a coronary vessel. Which of the following is the most likely diagnosis? A. Acute MI due to occlusion of left coronary artery B. Acute MI due to occlusion of right coronary artery C. Acute MI due to occlusion of circumflex artery D. Angina due to occlusion of left coronary artery E. Angina due to occlusion of left anterior descending artery

C. Acute MI due to occlusion of circumflex artery

Clicker question: A 28-year-old woman comes to the physician because of a 2-day history of a pruritic skin rash. She was recently prescribed a course of penicillin for an upper respiratory tract infection. Her temperature is 39°C (102.2°F); other vitals are WNL. PE shows generalized maculopapular rash and vesicular breath sounds. CBC shows eosinophilia; Urinalysis shows pyuria; Serum creatinine is 1.8 mg/dL (N: 0.7 to 1.2). Which of the following is the most likely diagnosis? A. Post-streptococcal GN B. TIN with Uveitis C. Allergic interstitial nephritis D. Obstructive nephropathy E. Papillary necrosis

C. Allergic interstitial nephritis

A 57-year-old man comes to the physician because of a 2-week history of palpitations, fatigue and light-headedness. He has 5-year history of coronary artery disease. Physical examination shows a pulse of 150/min. An ECG is shown in the figure. Which of the following is the most likely diagnosis? A. Sinus tachycardia B. Atrial tachycardia C. Atrial flutter D. Atrial fibrillation E. Third degree heart block F. Ventricular tachycardia

C. Atrial flutter

A 58-year-old man comes to the physician because of a 4-month history of shortness of breath, chest pain and syncope. Physical examination shows low volume pulse, sustained apical impulse and systolic murmur. Transesophageal echocardiogram shows stenosed bicuspid aortic valve. An ECG shows left axis deviation. Which of the following additional physical findings is most likely in this patient? A. Wide pulse pressure B. Opening snap in mitral area C. Audible S4 D. Hypertension E. Radio-femoral delay in peripheral pulse

C. Audible S4

A 61-year-old man comes to the physician for a follow up examination. He had an inferior wall MI 2 years ago. His vital signs are within normal limits. Physical examination shows bilateral basal crepitations. An ECG shows pathological Q waves in lead II, III & aVF. Which of the following additional physical findings most likely indicates ventricular remodeling in this patient? A. Pansystolic murmur in mitral area B. Parasternal heave C. Audible S4 in the apex D. Loud A2 in aortic area E. Tender hepatomegaly

C. Audible S4 in the apex

7. A 56-year-old woman comes to the physician because of a 2-week history of shortness of breath and swollen feet. She has a 15-year history of emphysema. Her pulse is 104/min, blood pressure is 98/64 mm Hg and respirations are 22/min. Physical examination shows elevated JVP, positive hepatojugular reflux, pitting ankle edema, sternal heave, loud S3 at the lower left sternal border and tender hepatomegaly. Echocardiography shows increased wall thickness of the right ventricle and interventricular septum deviating towards the left ventricle. Which of the following additional physical findings is most likely in this patient? A. Radial-femoral delay B. Pulsus bisferiens C. Bilateral basal rales D. Friction rub E. Absent breath sounds in lower lung fields

C. Bilateral basal rales

A 54-year-old male patient comes to his physician because of lower back pain and tiredness since the past 2 months. There is no history of hypertension or diabetes mellitus. Physical examination shows conjunctival pallor and pitting pedal edema. Laboratory studies show Hb 8.6 g/dL, serum creatinine of 1.9 mg/dL and serum calcium of 12.5 mg/dL. Peripheral smear shows normocytic normochromic RBCs and rouleaux formation. X- ray shows lytic lesions in the lumbar vertebrae and skull. Urine examination shows proteinuria 5.6 g/24 hours with light chain excretion. H and E stained picture of the renal biopsy is shown. Which of the following investigations will confirm the primary diagnosis in this patient? A. Congo red staining of the renal biopsy B. Flow cytometry of peripheral blood C. Bone marrow aspirate D. Immunofluorescence of renal biopsy E. Electron microscopy of renal biopsy

C. Bone marrow aspirate (multiple myeloma —> CRABS criteria)

Left-sided pressure recording obtained from a 24-year-old healthy young male is shown below. Which of the following alphabets most likely represents the closure of aortic valve? A. A B. B C. C D. D E. E F. X

C. C

9. A 39-year-old woman comes to the physician because of a 5-month history of shortness of breath. She had rheumatic fever 7 years ago. Physical examination shows a parasternal heave, a loud S1 in mitral area, a loud S2 in pulmonary area and bibasilar rales in lung fields. An ECG shows right axis deviation. Which of the following best explains a loud S1 in this patient? A. Elevated left ventricular EDV B. Calcified mitral valve leaflets C. Elevated left atrial pressure D. Left atrial hypertrophy E. Elevated LV afterload

C. Elevated left atrial pressure

A 67yo with Colon cancer undergoes resection followed by ileostomy. For ostomy losses Normal saline at 90 ml/hour compensates. Initially, their urine output is approximately 40 ml per hour but drops to 20 ml/hour on post-op day 4. An arterial blood gas is remarkable for a pH of 7.21 and PCO2 of 28 mmHg. Serum sodium is 130 mEq/L, potassium is 5.6 mEq/L, BUN is 90 mg/dL, and creatinine is 3.0 mg/dL. Urine sodium is 35 mEq/L, and urinary creatinine is 90 mg/L. Which of the following is the next most appropriate step in establishing the underlying pathology leading to deranged renal function? A. Renal biopsy B. Urine for dysmorphic RBCs C. Fractional excretion of sodium D. Ultrasound abdomen and pelvis E. Increase the IV fluid rate

C. Fractional excretion of sodium

A 66-year-old man comes to his physician because of progressively increasing swelling around ankles and feet, and around the eyes when he wakes up in the morning, since the past 3 months. He was diagnosed with hypertension 6 years ago and diabetes mellitus 10 years ago but is not very compliant with the prescribed medications. Physical examination shows bilateral pitting pedal edema and conjunctival pallor. On examination, the collected urine appears frothy. Laboratory investigations reveal a proteinuria of 5.6 g/24 hours with dysmorphic RBCs and RBC casts. Serological investigations show hypoalbuminemia and hypercholesterolemia. Which part of the kidney is involved here? A. Tubules B. Interstitium C. Glomerulus D. Renal artery E. Pelvicalyceal system

C. Glomerulus

A 35-year-old man comes to the physician because of a 2-day history of blood in urine following a 5-days history of mild flu like illness. He has had 6 episodes of similar complaints in last 4 years which used to subside on their own in 2-3 days. He has a past 5-year history of gluten sensitivity. His temperature is 98.4°F(37°C) and blood pressure is 150/96mm Hg. Urinalysis shows a pH of 7, 1+ proteinuria, 2+hematuria, and; no glucose, ketones, WBCs, casts, or crystals. Which of the following findings on immunofluorescence studies of the renal specimen is most likely in this patient? A. Linear IgG deposits along the GBM B. Coarse granular C3 and IgG deposits in the capillary loops C. Granular IgA deposits in the mesangium D. Absence of deposits E. Granular IgG, IgA, IgM deposits with C3, C1q in mesangium and capillary loops

C. Granular IgA deposits in the mesangium

The reason his FeNa will be >4% is because: A. He has a normal GFR B. He has over dilute urine C. His serum Creatinine is disproportionately elevated D. He has pre-renal AKI E. His FeNa is not >4%

C. His serum Creatinine is disproportionately elevated

Clicker question: A 3-year-old boy is referred to a specialist because of a 2-year history of growth delay. He has intermittent episodes of joint discomfort while walking and a few instances of falling while running. His mother also noticed increased thirst and urination for the past 6 months. He has a paternal uncle with kidney issues and photophobia. PE shows short stature, low muscle tone, and widening of the wrists, knees, and ankles. Which of the following lab findings is most likely expected in this patient? A. Metabolic alkalosis B. Hyperglycemia C. Hyperphosphaturia D. Hyperuricemia E. Hyperkalemia

C. Hyperphosphaturia

3. A 43-year-old man comes to the physician because of a 6-month history of shortness of breath. His symptoms were mild initially, but have worsened, even at rest. He emigrated from Asia 1 year ago. He has no history of any chest pain, syncope, or palpitations. He is a non-smoker and non-alcoholic. He was diagnosed with pulmonary tuberculosis 2 years ago and was treated with anti-tubercular drugs. His vital signs are all within normal limits. Physical examination shows elevated jugular venous pulse with a rapid and prominent "y" descent, bilateral ankle edema, and tender hepatomegaly. A chest X-ray shows pericardial calcifications. Which of the following additional physical findings is most likely in this patient? A. Muffled heart sounds B. Pansystolic murmur C. Increase in JVP during inspiration D. Pistol shot sound in femoral arteries E. Radial-femoral delay of arterial pulse

C. Increase in JVP during inspiration (pulses paradoxus secondary to cardiac tamponade)

9. A 70-year-old woman comes to the physician because of a 2-week history of shortness of breath on exertion. She has a 15-year history of hypertension. Her temperature is 37⁰C (98.7⁰F), pulse is 90/min, blood pressure is 170/94 mmHg, respirations are 18/min. Physical examination shows loud S2 in aortic area and audible S4 in mitral area. Lungs are clear. Which of the following left ventricular findings is most likely consistent with this patient? (EF=Ejection Fraction) Peak systolic Pr - Diastolic Pressure - EF A. Decreased - No change - Decreased B. Increased - Increased - Increased C. Increased - Increased - Normal range D. No change - Decreased - Decreased E. Increased - No change - Normal range

C. Increased - Increased - Normal range

A 53-year-old man with a 30 pack-years of cigarette smoking comes to the physician because of a 1-year history of shortness of breath. His pulmonary function studies show: Elevated total lung capacity and functional residual capacity, Low forced vital capacity and Low FEV1/FVC. These findings are most consistent with which of the following? A. Decreased lung compliance B. Decreased strength of the chest wall muscles C. Increased airway resistance D. Increased chest wall compliance E. Increased diffusion distance

C. Increased airway resistance

A 37-year-old woman comes to the physician because of a 4-month history of shortness of breath on exertion. Her vital signs are within normal limits. Physical examination shows a loud S1 and a mid-diastolic murmur in the mitral area and a parasternal heave. Cardiac catheterization of the left heart is performed to assess the severity of her cardiac condition. (Normal values: Aortic Pressure (AP): 120/80 mm Hg; Left ventricular pressure (LVP): 120/5 mmHg; Left atrial pressure (LAP): 8-12 mm Hg.)Which of the following combinations of pressure values (in mm Hg) is most likely in this patient? A. LVP of 120/80 and LAP of 22 B. LVP of 160/5 and LAP of 10 C. LVP of 200/10 and LAP of 26 D. LVP of 120/5 and LAP of 18 E. LVP of 100/40 and LAP of 20

C. LVP of 200/10 and LAP of 26 (mitral stenosis)

A 52-year-old woman comes to the emergency department because of a 1-hour history of weakness in her right upper and lower limbs and difficulty in speaking. She was diagnosed with mitral stenosis 5 years ago. Her pulse ranges between 58-94/min and has an irregularly irregular rhythm. Physical examination shows right-sided stroke and dysarthria. Which of the following best explains this patient's presentation? A. Insufficient left-sided cardiac output B. Left ventricular ectopics C. Left atrial fibrillation D. Arteriosclerosis in carotid arteries E. Insufficient filling of the left ventricle

C. Left atrial fibrillation

A 55-year-old man, is brought to the emergency department because of a 30-min history of intense substernal chest pain and vomiting. He has a 30-year history of smoking and a 10-year history of type 2 diabetes mellitus. An ECG taken on arrival shows ST segment elevation. Laboratory studies show elevated cardiac markers. Which of the following morphologic changes is most likely in the injured myocardium of this patient? A. Coagulation necrosis B. Granulation tissue formation C. Mitochondrial swelling D. Macrophage invasion E. Scar tissue

C. Mitochondrial swelling

A 56-year-old woman comes to the physician because of a 2-week history of nervousness and anxiety. She had total thyroidectomy done 8 months ago and is taking thyroid supplement. Physical examination shows no abnormalities. An ECG in lead II is shown below. Which of the following ECG findings is most likely in this patient? A. P waves more than QRS waves B. A wide QRS interval C. R-R interval less than 0.6 sec D. Inverted P waves in lead II E. Absent T waves and appearance of U waves F. PR interval >5 small boxes

C. R-R interval less than 0.6 sec

A 17-year-old boy comes to your clinic for evaluation of 3 days of "dark urine." Of note, he recently competed in a local marathon. He had previous episodes of dark urine, all following strenuous exercise, which resolved without intervention. Physical examination reveals an alert, thin boy in no respiratory distress. Blood pressure and heart rate are normal for age. The remainder of the physical examination is benign with the exception of eczematoid rash in the antecubital fossa bilaterally. Urinalysis reveals the presence of blood and protein with many red blood cells. Which of the following is the most likely diagnosis? A. Alport syndrome B. IgA vasculitis C. Rhabdomyolysis D. Exercise-induced hematuria E. IgA nephropathy

C. Rhabdomyolysis

A 65-year-old-female is brought to the emergency because of a 12-hour history of acute right sided abdominal pain that radiates to the groin. The pain comes in waves, and she cannot find a comfortable position. She also has a 12-month history of 3 episodes of UTI and one hospitalization for acute pyelonephritis of the right kidney. Her temperature is 100°F(37.7°C), pulse is 95/min, respirations are 16/min and blood pressure is 130/80mm Hg. Urinalysis shows a pH of 8.0, leukocyte esterase positive, nitrite positive, WBC 70-90/HPF, RBC 30-40/HPF. Coronal non-contrast CT image is shown.Which of the following is the most likely stone type ? A. Calcium phosphate B. Uric acid C. Struvite D. Cystine E. Calcium oxalate

C. Struvite

A 35-year-old female comes to the physician because of a 3-day history of generalized edema. She also has a 6-months history of facial rash, arthralgias and recurrent oral ulcers. Her pulse is 100/min and blood pressure is 146/90mm Hg. Physical examination shows an erythematous rash on the cheeks (in fig), 3+ pedal edema and joint swelling in the metacarpophalangeal joints. Urinalysis shows fatty casts, RBC casts and elevated serum creatinine and decreased GFR. 24-hour urine shows 3.8g of protein/day. Light microscopy of renal biopsy shows diffuse capillary proliferation. The patient is started on corticosteroids, and IV cyclophosphamide. Which of the following serological tests will most likely show effectiveness of the response to immunosuppressive therapy? A. ANA Ab B. Anti -Smith Ab C. dsDNA Ab D. Antiphospholipid Ab E. SSA

C. dsDNA Ab

A 26-year-old woman comes to the physician because of a 3- week history of increased thirst and frequency of urination especially at night. She constantly feels thirsty and drinks water every half hour. She was in a roadside accident about a month ago, where she had head injury. Her blood pressure is 120/76mm Hg, pulse is 82/min and respirations are 12/min. Laboratory studies show:Blood glucose: 82 mg/dL (N= 70-100 mg/dL)Serum sodium: 156 mEq/L (N= 136-146 mEq/L)Serum potassium: 4.0 mEq/L (N=3.5-5.0 mEq/L )Bicarbonate: 23 mEq/L (N=22-28 mEq/L)Which of the following sets of changes will be most likely seen in the renal tests? Urine osmolality Plasma Renin Urine volume A. ↔ ↓ ↑ B. ↔ ↑ ↑ C. ↓ ↑ ↑ D. ↓ ↔ ↔ E. ↑ ↓ ↔

C. ↓ ↑ ↑

A 28-year-old man with a long-standing history of severe asthma presents to the emergency room with shortness of breath. He has previously required admission to the hospital and was once intubated for asthma. Which of the following findings on physical examination would predict a mild course of asthma? A. Silent chest B. Hypercapnia C. Thoracoabdominal paradox D. 5mmHg drop in systolic BP during inspiration E. Altered mental status

D. 5mmHg drop in systolic BP during inspiration

An ECG of a 26-year-old healthy normal man is shown below. Which of the following values is the most likely heart rate of this person?(one small box = 0.04 sec) A. 150 B. 100 C. 90 D. 75 E. 60

D. 75

42-year-old woman comes to the physician for a routine examination. Her pulse is 100/min and blood pressure is 126/82 mm Hg. Physical examination shows no abnormalities. Laboratory studies show: O2 content of arterial blood=20 mL/dL; O2 content of pulm.arterial blood=16 mL/dL; Oxygen consumption=300 mL/min. Her Left ventricle PV loop is shown in the figure.Which of the following is the most likely cardiac output in her? A. 3500 mL/min B. 4000 mL/min C. 6500 mL/min D. 7500 mL/min E. 8000 mL/min

D. 7500 mL/min (CO = SV x HR)

A 36-year-old previously healthy woman is brought to the emergency department because of a 6-hour history of a severe crush injury to her right lower limb while on the job at construction site. A pinning and reconstructive surgery is performed, and perioperative broad-spectrum antibiotics are started. Her blood pressure remained normal throughout her hospital course. Her laboratory studies show: Hb 10.2g/dl (12-15 g/dl), HCT 32% (36%-47%), BUN 40mg/dl (8-21 mg/dl), Creatinine 3.8mg/dl (0.6-1.2 mg/dl), and; S. Creatinine kinase 3600 U/L (25-200 U/L). Her urine output dropped to 0.3 mL/kg/h. His FENa is 2.5%. Which one of the following is the most likely cause of AKI in this patient? A. Prerenal B. Acute interstitial nephritis C. Post renal D. Acute tubular necrosis E. Vascular

D. Acute tubular necrosis

Clicker question: The patient is diagnosed with stage G2 chronic kidney disease. Reviewing his vital signs show a blood pressure of 138/85 mmHg; other vitals WNL. His HbA1c is 7.0%. His eGFR is 80 ml/min/1.73 m2. Initiation of antihypertensive therapy is planned. Which of the following classes of medications is the most appropriate initial antihypertensive therapy for this patient? A. Alpha-2 agonists B. Beta-blockers C. Calcium channel blockers D. Angiotensin receptor blockers E. Vasodilators

D. Angiotensin receptor blockers

Based on the physical findings, which of the following valvular heart diseases is most likely in the previous patient? A. Mitral stenosis B. Aortic stenosis C. Mitral regurgitation D. Aortic regurgitation E. Pulmonary stenosis F. Pulmonary regurgitation G. Tricuspid stenosis H. Tricuspid regurgitation

D. Aortic regurgitation

A 69-year-old man comes to the physician because of a 6-month history of shortness of breath, chest pain and syncope. Physical examination shows low volume pulse, sustained apical impulse and an ejection systolic murmur in the left sternal border in the third intercostal space. An ECG shows left axis deviation. Which of the following structural changes is most likely in this patient's heart? A. Hypertrophy of the interventricular septum B. Eccentric hypertrophy of the left ventricle C. Eccentric hypertrophy of the right ventricle D. Concentric hypertrophy of the left ventricle E. Concentric hypertrophy of the right ventricle

D. Concentric hypertrophy of the left ventricle

A 65-year-old man with a 20-year history of COPD comes to the physician because of sudden onset of dyspnea. PE Show tachycardia, ankle edema, elevated JVP, positive hepatojugular reflux, tender hepatomegaly, and bibasilar rails in the lower lung fields. Which of the following is the most likely the cause of his new dyspnea? Session ID: sidowu A. Elevated systemic blood pressure B. Left ventricular hypertrophy C. Decreased right ventricular preload D. Decreased left ventricular preload E. Elevated pulmonary arterial pressure

D. Decreased left ventricular preload

A 9-year-old girl comes to the physician because of a 5-day history of nausea, vomiting, passing "cola-colored" urine and generalized weakness. She also has a 5-week history of recovering from honey-colored crusts on her face. Her temperature is 97.8 °F (36.5°C), pulse is 90/min and blood pressure is 150/90 mm Hg. Physical examination shows periorbital edema. Laboratory studies show BUN/Creatinine ratio of 9 and fractional sodium excretion of 2.5%. Urinalysis shows 2+ hematuria, 2+proteins, 12-14/HPF RBCs with dysmorphic features and numerous RBC casts. Which of the following will be most likely seen on light microscopy? A. Glomerular tram track appearance B. Normal-appearing glomeruli C. Mesangial hypercellularity with expansion of the mesangial matrix D. Global and diffuse hypercellular glomeruli and neutrophilic infiltration E. Focal and segmental hypercellular glomeruli and neutrophilic infiltration

D. Global and diffuse hypercellular glomeruli and neutrophilic infiltration

Clicker question: Which of these medications may be safely used during pregnancy? A. Telmisartan B. Spironolactone C. Captopril D. Labetalol E. Aliskiren

D. Labetalol

2. A 56-year-old woman is brought to the emergency department because of a 20-min history of crushing chest pain that started while she was jogging in the park. Her pulse is 124/min and blood pressure is 112/66 mmHg. An ECG recorded ion her arrival shows ST segment elevation in leads I, aVL, V5 and V6. After stabilization, a coronary angiography is performed that shows left coronary dominance and a block in one of the major coronary arteries. Which of the following coronary vessels is most likely blocked in this patient? A. Right coronary artery B. Left coronary artery C. Posterior descending artery D. Left circumflex artery E. Left anterior descending artery

D. Left circumflex artery

A 69-year-old man comes to the physician because of a 4-week history of light-headedness. He does not have any major illnesses. His pulse is 48/min with regular rhythm and blood pressure is 110/72 mm Hg. Physical examination and laboratory studies show no abnormalities. Which of the following ECG findings most likely suggests a cardiac arrhythmia in this patient? A. Tall T waves B. ST segment depression C. Pathologic Q wave D. Long R-R intervals E. Tall QRS complexes in V5,6 F. ST segment elevation

D. Long R-R intervals

10. A 37-year-old man comes to the physician because of a 3-month history of dyspnea, chest pain and episodes of syncope. He had rheumatic carditis 5 years ago. His pulse is 74/min and has low volume, and blood pressure is 116/70 mm Hg. Physical examination shows an ejection systolic murmur in the right second intercostal space close to the sternum and an audible S4 in mitral area. Echocardiography shows concentric left ventricular hypertrophy. Which of the following factors is most likely increased in this patient? A. End diastolic volume of the LV B. End systolic volume of the LV C. Stroke volume D. Peak LV pressure during ejection E. Peak aortic pressure during ejection

D. Peak LV pressure during ejection

6. A 64-year-old woman comes to the physician because of a 2-month history of chest discomfort and dyspnea. She received chemotherapy for breast cancer 8 month ago. Her pulse is 98/min, BP is 128/86 mmHg. Physical examination shows elevated JVP, positive hepatojugular reflux, audible S3, bibasilar rales in lower lung fields and tenderness below right costal margin. An x-ray of the chest shows bat wing's density and obliteration of costophrenic angles. Elevated level of which of the following biomarkers is most likely used to confirm the diagnosis? A. Plasma renin B. Serum CK-MB C. Total plasma cholesterol D. Plasma beta-natriuretic peptide E. Serum aldosterone

D. Plasma beta-natriuretic peptide

Physical examination of a 41-year-old man with a 7-year history of mitral stenosis of rheumatic etiology shows a mid-diastolic murmur in the mitral area. Which of the following features best characterizes this murmur? A. Murmur originates in the left atrium B. Best heard with the diaphragm of the stethoscope C. Murmur disappears when the patient develops atrial fibrillation D. Presystolic accentuation of the murmur indicates stronger atrial contraction E. Duration becomes shorter with severe stenosis

D. Presystolic accentuation of the murmur indicates stronger atrial contraction

11. A 65-year-old man is brought to the emergency department because of a 3-hour history of shortness of breath and confusion. He has a 15-year history of hypertension and a 2-year history of chronic kidney disease requiring hemodialysis. His temperature is 37°C (98.6°F), pulse is 122/min and blood pressure is 84/60 mm Hg. Physical examination shows elevated JVP with absent y descent and muffled heart sounds. An ECG shows low voltage waves and electrical alternans. Which of the following additional physical examination findings is most likely? A. Canon a waves in JVP B. Radiofemoral delay C. Pulsus bisferiens D. Pulsus paradoxus E. Warm extremities

D. Pulsus paradoxus (cardiac tamponade as an obstructive shock)

A 45-year-old man comes to his physician because of increasing shortness of breath, cough in blood and swelling around his eyes and ankles since the past 1 month. He also reports a weight loss of 6 kgs (13 pounds) in 2 months. The patient has been a chronic smoker since the age of 20 years. There is no history of hypertension or diabetes mellitus. Physical examination shows conjunctival pallor and clubbing of fingers. Respiratory examination reveals decreased breath sounds. Urine is positive for proteinuria and is quantified as 4.5 g/24 hours. Oval fat bodies are also seen in the urine sediment. A high-resolution CT scan of the chest shows a necrotic mass measuring 3 x 2 cms near the hilum of the left lung. Biopsy from the lesion confirms squamous cell lung carcinoma. A renal biopsy is also performed and shows a thickened glomerular basement membrane. Electron microscopy shows effacement of podocytes and subepithelial immune complex deposits. Which of the following is a possible complication of the renal disease in this patient? A. Horner's syndrome B. Cushing's syndrome C. Hypercalcemia D. Renal vein thrombosis E. Polycythemia

D. Renal vein thrombosis

Clicker question: A 65-year-old woman is brought to the ED for shortness of breath. She has 3-pillow orthopnea, bilateral ankle swelling, and a productive cough. She is on enalapril. Her heart rate is 105/min, irregularly irregular, RR: 25/min, BP: 150/95 mmHg. PE shows (+) JVD, PMI 6th ICS LMCL, with S3 gallop; (+) bibasal crackles, 2+ pitting bipedal edema. EKG shows atrial fibrillation.Which of the following drugs will most likely cause added angioedema when given to this patient? A. Metoprolol B. Spironolactone C. Valsartan D. Sacubitril E. Diltiazem

D. Sacubitril

A 43-year-old woman comes to the physician because of a 2-month history of weight gain, heat intolerance and hoarseness. She was diagnosed with Hashimoto's disease 6 months ago. Her ECG in lead II is shown below (rate less than 60 bpm). Which of the following is the most likely diagnosis? A. 1st degree heart block B. 2nd degree heart block-type1 C. 3rd degree heart block D. Sinus bradycardia E. 2nd degree heart block-type2

D. Sinus bradycardia

Clicker question: A 40-year-old woman comes to the physician for a follow-up. She is currently asymptomatic. She has a high-fat + high-salt diet and a sedentary lifestyle. She has a 10 pack-year history of cigarette smoking. Her PR is 85/min, RR is 18/min, BP is 138/85 mmHg; BMI is 30.46 kg/m 2. Her blood pressure on last week's check-up was 135/83 mmHg. What is the patient's blood pressure category? A. Normal B. Prehypertension C. Elevated D. Stage 1 E. Stage 2

D. Stage 1

A 24-year-old man is brought to the emergency because of a 3-hour history of severe, colicky right flank pain, urinary frequency and urgency. Physical examination shows right-sided costovertebral angle tenderness. Urine dipstick is heme positive. Urinalysis shows crystals as shown in fig. A non-contrast abdominal CT shows a renal stone. What is the most likely pathological basis for this condition? A. Supersaturation of the urine with citrate B. Infection with urease positive organisms C. Acidification of the urine D. Supersaturation of the urine with oxalate E. Increased free water clearance

D. Supersaturation of the urine with oxalate

A 42-year-old man comes to the physician because of a 3-week history of fever, chills and weight loss. He had a dental cleaning 3 weeks ago. He had an acute rheumatic fever at the age of 15. Physical examination shows splinter hemorrhages in his fingernails and a loud systolic murmur in the mitral area. Laboratory studies show positive blood culture for Gram-positive cocci in long chains. Which of the following pathogens is the most likely cause of his condition? A. Staphylococcus aureus B. Enterococcus faecalis C. Hemophilus aphrophilus D. Viridans streptococci E. Staphylococcus epidermidis

D. Viridans streptococci

11. A 24-year-old man comes to the physician because of a 2-month history of palpitations. He says that he experiences episodes of palpitations while swimming and playing basketball. His brother died of a heart disease at the age of 20. His vital signs are normal. Physical examination shows no abnormalities. An ECG at rest is shown in the figure. Which of the following is the most likely diagnosis? A. Angina pectoris B. Anterior wall MI C. Long QT syndrome D. WPW syndrome E. Brugada syndrome

D. WPW syndrome

A 24-year-old man comes to the emergency department because of a 2-hour history of recurrent episodes of palpitations. He says he had similar episodes for the last 6 months but the frequency of these has increased recently after he started drinking large amount of coffee. He is hemodynamically stable. An ECG taken in lead II during an episode is shown in fig A. He is given a 12 mg bolus dose of adenosine and the ECG taken is shown in fig B. Which of the following is the most likely diagnosis? A. Sinus Tachycardia (ST) B. First degree heart block C. WPW syndrome D. Atrial tachycardia (AT) E. AV-nodal reentry tachycardia (AVNRT)

E. AV-nodal reentry tachycardia (AVNRT)

A 5-year-old girl is brought to her pediatrician because of swelling around eyes especially in the morning, since the past 3 weeks. The mother says the swelling gets better as the day progresses. The patient had sore throat and mild fever 4 weeks back but resolved with home remedies. She has been vaccinated as per schedule. Physical examination shows mild pedal edema which is pitting in nature. Urine examination shows proteinuria of 9.8 g/24 hours. Oval fat bodies and fatty casts are observed on microscopic examination of the urinary sediment. There are no RBCs or RBC casts in urine. Which of the following investigations will reveal the underlying characteristic pathology? A. Renal biopsy stained with H and E B. Renal biopsy stained with PAS stain C. Immunofluorescence with IgG D. Immunofluorescence with C3 E. Electron microscopy

E. Electron microscopy

Physical examination of a 36-year-old woman with a 6-year history of mitral stenosis of rheumatic etiology shows evidences of pulmonary hypertension and right ventricular hypertrophy. Which of the following hemodynamic alterations most likely initiated the pathogenesis of these changes? A. Increased blood flow through the mitral valve B. Recurrent pulmonary embolisms C. Increased left ventricular compliance D. Pulmonary vascular remodeling E. Elevated left atrial pressure

E. Elevated left atrial pressure

A 35-year-old woman comes to the physician because of a 3-month history of episodic palpitations and diaphoresis. Physical examination shows pallor and hypertension. An ECG taken during an episode is shown in the figure. Laboratory studies show increased levels of urine and serum catecholamines, metanephrines, and normetanephrines. Which of the following is the most likely mechanism of genesis of this patient's tachyarrhythmia? A. Triggered activity of atrial muscle B. Triggered activity of the sinoatrial node C. Generation of a reentrant circuit in atria D. Generation of a reentrant circuit in AV node E. Enhanced automaticity at the sinoatrial node F. Enhanced automaticity in the Purkinje fibers

E. Enhanced automaticity at the sinoatrial node

A 45-year-old man comes to physician for a follow up examination. He has a 10-year history of COPD. Which of the following combinations of lung volumes/capacities makes the total lung capacity (TLC)? A. FRC, IRV and ERV B. RV, Tidal vol and ERV C. RV, ERV and IRV D. Tidal vol, ERV and IRV E. FRC, Tidal vol and IRV

E. FRC, Tidal vol and IRV

Clicker question: A 55-year-old man comes to the clinic for follow-up. He is currently asymptomatic. He has a 15-pack year history of cigarette smoking and drinks 2 glasses of wine daily x 10 years. His BP is 135/85 mmHg; other vitals are within normal limits (WNL). PE and blood tests are unremarkable. He is advised lifestyle changes (smoking cessation, moderate alcohol consumption) and started on an antihypertensive drug. He returns 3 months later with a fasting blood glucose of 116 mg/dL. Which of the following is most likely expected with continued use of the antihypertensive drug? A. Hyperkalemia B. High HDL and low LDL C. Hypocalcemia D. Low risk of gout E. Hypomagnesemia

E. Hypomagnesemia

A 72-year-old man comes to the emergency department because of a 2-day history of wheezing and shortness of breath. He has a 24-year history of COPD. Physical examination shows tachycardia, irregular pulse, loud P2, hyperresonant percussion notes and rhonchi all over the lung fields bilaterally. His ECG in lead II is shown below. Which of the following is the most likely diagnosis? A. Sinus tachycardia B. Focal atrial tachycardia C. Polymorphic ventricular tachycardia D. Monomorphic ventricular tachycardia E. Multifocal atrial tachycardia F. Atrial fibrillation

E. Multifocal atrial tachycardia

Indicate the 4 main types of shock 1. Hypovolemic 2. Anaphylactic 3. Distributive 4. Electrical 5. Cardiogenic 6. Emotional 7. Obstructive 8. Cold

1,3,5,7

9. A 50-year-old man comes to the clinic because of cough for the past one month. He says that cough is productive in nature with copious amount of thick sputum more during the morning hours. The cough has been there for 4 years but worsened for the past one month. He has a history of smoking 1 pack of cigarette daily for 30 years. Which of the following is a most likely radiological feature in the above condition? 1. A. Hyperlucency 2. B. Increased X-ray markings 3. C. Retrosternal airspace 4. D. Bullous changes 5. E. Low flat diaphragms

2. B. Increased X-ray markings

13. A 14-year-old girl is brought to the emergency department by her mother because of a persistent cough and wheezing that began 2 hours ago. Her sister has eczema, and her mother has allergic rhinitis. Her temperature is 37°C (98.6°F), pulse is 108/min, blood pressure is 110/80 mm Hg, respirations are 25/min, and oxygen saturation is 88% on room air. Bilateral wheezing is heard on auscultation of the chest, and the patient is using accessory respiratory muscles. Which of the following inflammatory mediators are most likely responsible for eosinophil recruitment in this patient's condition? 1. A. IL 1 2. B. Matrix metalloproteinase 3. C. Bradykinin 4. D. IL 5 5. E. Histamine

4. D. IL 5

A 3-year-old girl presents to the emergency department with purple skin rashes in her legs. She had an episode of viral sore throat 3-weeks ago. Physical examination shows multiple non-blanching, purple papule and petechiae in both the legs. The complete blood count demonstrates a low platelet count, while the peripheral blood smear is notable only for large platelets. Which of the following laboratory findings would most likely be present in this patient? A. Antiplatelet antibodies B. Decreased megakaryocytes on bone marrow biopsy C. IgA deficiency D. Increased fibrin split products E. Vitamin K deficiency

A. Antiplatelet antibodies

11. A 36-year-old woman comes to the physician because of a 1-week history of wheezing and shortness of breath. She reports "sniffling" more frequently, feels congested, and has a clear nasal discharge. She has been taking aspirin for back pain for the past 2 weeks. She takes losartan for hypertension. Physical examination shows a polyp in the right nostril. Diffuse rhonchi are heard on lung auscultation.Which of the following is the most likely cause of this patient's condition? A. Aspirin B. Dust mite allergy C. Lisinopril D. Seasonal allergies E. Viral upper respiratory tract infection

A. Aspirin

Based on the above formula, increase in which of the following factors most likely cause pulmonary hypertension? (you may choose more than one response) A. High cardiac output B. Increased pulmonary vascular resistance C. Elevated red cell mass (hematocrit) D. Increased systemic arterial pressure E. Elevated left atrial pressure F. Tricuspid stenosis G. Hypovolemia H. Pulmonary stenosis

A. High cardiac output B. Increased pulmonary vascular resistance C. Elevated red cell mass (hematocrit) D. Increased systemic arterial pressure E. Elevated left atrial pressure

A 60-year-old man with multiple co-morbidities including COPD, heart failure, type 2 diabetes mellitus and recurrent pneumonias, comes to the physician after a recent hospitalization for COPD exacerbation. He also has a history of chemoradiation for prostate cancer 2 years back. He is complaining of fever and fatigue. His temperature is 97.4°F(36.3°C) and oxygen saturation is 90%. Physical examination shows wheezing on auscultation. Laboratory tests show leukocytosis with a left shift with a total WBC count of 56,000/mm3(4,500-11,000/mm3). A differential count shows 63% segmented neutrophils, 16% bands, 7% metamyelocytes, 3% myelocytes, 1% blasts, 8% lymphocytes, and 2% monocytes. PSA levels are normal. Karyotype shows a normal male karyotype (46,XY). Which of the following is most likely associated with this patient's condition? A. High leukocyte alkaline phosphatase B. Presence of smudge cells on peripheral blood film C. Elevated D-dimer D. Positive TdT assay E. Low leukocyte alkaline phosphatase

A. High leukocyte alkaline phosphatase

7. A 58-year-old man is admitted to the hospital because of a 2-day history of fever, chills, cough and dyspnea. She is diagnosed with extensive air-space consolidation in the left upper and lower lobes due to Streptococcus pneumoniae. On the second day of hospitalization, he develops extreme shortness of breath when positioned with his left side down. An arterial blood sample drawn in this position shows PaO2 of 46 mmHg. An arterial blood sample drawn while his right side was down had showed a PaO2 of 74 mmHg. Which of the following best explains the drop in PaO2 when the man is positioned on his left side down? A. Increased blood flow to the diseased lung B. Increased ventilation to the normal lung C. Increased airway resistance in the normal lung D. Formation of interstitial edema in the diseased lung E. Increased stiffness of the chest wall on the diseased side

A. Increased blood flow to the diseased lung

A 4-year-old girl is brought to the physician because of a 1-week history of fever. She is also less energetic and walks with a limp. Physical examination shows hepatomegaly, scattered petechiae, and bruising over many surfaces of her body. Laboratory tests show : Hemoglobin: 5.9 g/dl (11.5-15.5 g/dl ) WBC count: 26,000/mm3 (4,500-14,500/ mm3) Platelet count: 39,000/mm3 (150,000-400,000/ mm3)Peripheral smear and flow cytometry as shown: Which of the following statements are true about the diagnosis?( Can select multiple options) A. It is an acute B-cell leukemia B. It is an acute B-cell lymphoma C. Pancytopenias are very common presenting in days to weeks D. Neoplastic cells proliferate slowly, pancytopenias in months to years E. Aggressive course, fatal, respond to chemotherapy.

A. It is an acute B-cell leukemia C. Pancytopenias are very common presenting in days to weeks E. Aggressive course, fatal, respond to chemotherapy.

23-year-old man brought to the ER with penetrating wound to the chest wall. Which of the following is the most likely diagnosis? A. Large Pneumothorax B. Small Pneumothorax C. Large Hemothorax D. Small Hemothorax E. Musculoskeletal Injury

A. Large Pneumothorax

Patient 1: A 38-year-old woman comes to the physician because of a 6-month history of dyspnea both at rest and during exercise and swelling in her legs and feet. Initially she had vague symptom of easy fatigue. Her respirations are 20/min and other vital signs are within normal limits. Physical examination shows distended jugular vein, tender hepatomegaly, ascites and pitting edema in her legs. Cardiac auscultation shows Loud P2 component of second heart sound and an audible fourth heart sound over tricuspid area. Laboratory studies of urine and blood show no abnormalities. Which of the following differentials could be considered at this point? (you may choose more than 1 answer) A. Right heart failure B. Renal failure C. Liver cirrhosis D. Left heart failure E. Acute myocardial infarction F. Pneumonia G. Valvular heart disease

A. Right heart failure G. Valvular heart disease

8. A 65-year-old man is brought to the emergency because of a 3-day history of productive cough and high fever. He has an 8-month history of low back pain. His temperature is 39.5°C (103.1°F), pulse is 95/min, respirations are 24/min, and blood pressure is 135/90 mmHg. Laboratory studies show: Hemoglobin 10.8 g/dL (12-16g/dL) Serum Calcium 13mg/dl (8.4-10.2 mg/dl) Serum Creatinine 1.5mg/dl (0.6-1.2mg/dl) Serum Albumin 3.8 g/dl (3.5-5.5)Gram staining of the sputum shows numerous Gram -positive diplococci. A chest x-ray shows right lower lung consolidation. His condition deteriorates and he dies. On autopsy, his kidneys are firm and pale. There is abundant pink hyaline material (in the glomeruli and atrophic tubules) which stains positively with Congo red. Which of the following additional findings was most likely to be detected one week prior to his death? A. Rouleaux formation in peripheral smear B. Increased erythropoietin levels C. Venous thrombi in circulation D. Blastic bone lesions E. Megaloblasts in bone marrow

A. Rouleaux formation in peripheral smear Multiple myeloma

2. A 63-year-old man comes to the physician because of a 2-week history of 2 episodes of hemoptysis and a 3-month history of dry cough, weight gain of 5 kg (12 lbs.) and skin pigmentation. He has a 40 pack-year history of cigarette smoking. His temperature is 37°C (98.6°F), pulse is 82/min and blood pressure is 166/100 mmHg. Physical examination shows truncal obesity, hyperpigmentation of the skin and abdominal striae. An x-ray of the chest shows a 5-cm right hilar mass. Histologic examination of a transbronchial biopsy of this mass shows clusters of small monochromatic cells with scant cytoplasm. Which of the following types of lung cancer is most likely? A. Small cell lung carcinoma B. Squamous cell carcinoma C. Adenocarcinoma D. Bronchioloalveolar carcinoma E. Large cell carcinoma

A. Small cell lung carcinoma (seen with Cushings and SIADH D/t inc ACTH and inc ADH)

A 12-year-old boy comes to the physician because of a 6-day history of shortness of breath and easy bruising. He also has a 2-week history of fatigue and abdominal discomfort. Physical examination shows generalized lymphadenopathy. X-ray of the chest shows large anterior mediastinal mass. A biopsy specimen of the mass is obtained and microscopically shows sheets of immature lymphoid cells. PBF and CBC is as shown. Immunophenotyping of the cells show terminal deoxynucleotidyl transferase (Tdt), CD2, CD3, and CD7. What is the most likely diagnosis? A. T-lymphoblastic leukemia/lymphoma (T-LBLL) B. Hodgkin lymphoma C. Thymoma D. Hairy cell leukemia E. B-lymphoblastic leukemia/lymphoma (B-LBLL)

A. T-lymphoblastic leukemia/lymphoma (T-LBLL)

10. A 59-year-old man is admitted to the hospital because of an acute exacerbation of COPD. He is found to be in acute respiratory failure and requires intubation with mechanical ventilation. He has a dead space (VD) of 100 mL. Which of the following methods of ventilation is most appropriate to increase PaO2 and decrease PaCO2 in this patient? A. Tidal volume of 700 mL and frequency of 10 breaths per minute B. Tidal volume of 500 mL and frequency of 12 breaths per minute C. Tidal volume of 400 mL and frequency of 14 breaths per minute D. Tidal volume of 700 mL and frequency of 6 breaths per minute E. Tidal volume of 200 mL and frequency of 25 breaths per minute

A. Tidal volume of 700 mL and frequency of 10 breaths per minute

A 4-year-old boy is brought to the physician because of a 5-day history of rash on lower legs. He also has a 5-week history lethargy and an enlarging left sided neck mass. Physical examination shows diffuse bilateral lower limb petechial hemorrhages and a non-tender enlargement of the left posterior cervical lymph node. A lymph node biopsy shows a predominance of interfollicular infiltrate containing numerous blast cells and focal necrosis. Immunophenotyping shows positive CD10, CD19 and Tdt. Which of the following translocations is most likely associated with a good prognosis? A. t(12;21) B. t(14;18) C. t(8;14) D. t(9;22) E. t(15;17)

A. t(12;21)

A 65-year-old man comes to the physician because of a 1-month history of easy fatiguability and widespread ecchymosis. His temperature is 37.2°C (99°F) and blood pressure is 130/80mm Hg. Physical examination shows enlargement of liver and spleen. Stool sample is positive for occult blood. Laboratory tests show hemoglobin 9.0g/dl (13.5-17.5 g/dl), platelets 16,000/mm3 (150,000-400,000/mm3),and WBCs 80,000 (4,500-11,000/mm3). Plasma D-dimer, fibrin degradation products and prothrombin time is high. Peripheral blood smear is as shown. Which of the following is the most likely diagnosis? A. Acute lymphoblastic leukemia B. Acute myeloid leukemia C. Chronic lymphocytic leukemia D. Chronic myeloid leukemia E. Hemophagocytic lymphohistiocytosis

B. Acute myeloid leukemia

6. A 74-year-old man is brought to the physician because of a 4-week history of fever, chills and productive cough. His sputum has a putrid odor. He was diagnosed with motor neuron disease 6 years ago and has difficulty swallowing for the last 4 months. His temperature is 39°C (102°F), pulse is 102/min and respirations are 22/min. Physical examination shows poor dental hygiene, increased vocal fremitus and amphoric breath sounds in the right lower lung base. An x-ray of his chest shows well defined thick walled cavitary lesion in the right lower lobe with air fluid levels as shown in the figure. Which of the following mechanisms most likely initiates his condition? A. Superinfection of a pulmonary cyst B. Aspiration of oropharyngeal secretion C. Septic embolization during bacteremia D. Direct extension of a mediastinal abscess E. Bronchial obstruction by a tumor mass

B. Aspiration of oropharyngeal secretion -aspiration PNA because of ALS -lung abscess

7. A 57-year-old woman comes to the physician for a routine examination. She has no personal or family history of major medical illnesses. Laboratory studies show: Hemoglobin 9.1 g/dl (13.5-17.5. g/dl); Platelet count 90,000/mm3 (150,000-400,000/mm3); Total leukocyte count 95000/mm3 (4,500-11,000/mm3). A photomicrograph of a peripheral blood smear is shown.Which of the following additional findings is most likely seen in this patient? A. Erythromelalgia B. Auto immune hemolytic anemia C. Disseminated intravascular coagulation D. Gum hypertrophy E. Blast crisis

B. Auto immune hemolytic anemia Smudge cells present so think CLL and spherocytes also present Erythromelagia in PCV DIC in APML Gum hypertrophy AML 5 Blast crisis CML

Karyotypic analysis shows the Ph1 chromosome, and molecular testing shows BCR-ABL fusion gene. The patient undergoes chemotherapy with imatinib mesylate (tyrosine kinase inhibitor) and achieves a complete remission but is then lost to follow-up. Three years later he develops fatigue, bone pains and 8-kg weight loss and appears again at the clinic. CBC shows : hemoglobin, 10.5 g/dl (13.5-17.5 g/dl); hematocrit, 30% (41%-53%); platelet count, 60,000/ mm3 (150,000-400,000/mm3) and WBC count, 40,000/mm3 (4,500-11,000/mm3). Flow cytometric analysis of the peripheral blood demonstrates CD19+, CD10+, sIg−, and CD3− cells. Which of the following complications of the initial disease did this patient most likely develop? A. Acute myeloid leukemia B. B-cell lymphoblastic leukemia C. Hairy cell leukemia D. Myelodysplastic syndrome E. Myelofibrosis

B. B-cell lymphoblastic leukemia

A 65-year-old man comes to the physician because of a 10-month history of gradual weight loss, weakness, anorexia, and easy fatigability. Physical examination shows marked splenomegaly. CBC shows: Hemoglobin, 11.9 g/dl (13.5-17.5. g/dl); Hematocrit, 39.1% (41%-53%); MCV, 92 µm3 (80-100 µm3); Platelet count, 589,000/mm3 (150,000-400,000/mm3); and WBCs, 145,000/mm3 (4,500-11,000/mm3). PBF is shown. The BM aspiration and biopsy reveal a markedly hypercellular BM and a markedly increased myeloid : erythroid ratio of 29:1 What is the most likely diagnosis? A. Chronic lymphocytic leukemia B. Chronic myeloid leukemia C. Essential thrombocytosis D. Acute myeloid leukemia E. Acute lymphoblastic leukemia

B. Chronic myeloid leukemia

12. A 22-year-old man with asthma comes to the clinic because of a chronic, productive cough that worsens at night. He also notes increased shortness of breath that respond to his albuterol inhaler. He does not use tobacco. Diffuse expiratory wheezing is heard on pulmonary auscultation. X-ray of the chest reveals hyperinflated lungs without evidence of focal consolidation. Which of the following would most likely be seen on pulmonary function testing prior to treatment? A. Decreased vital capacity B. Decreased FEV1/FVC C. Decreased residual volume D. Decreased total lung capacity E. Increased FEV1

B. Decreased FEV1/FVC

13. A 67-year-old woman comes to the physician for pre-surgical clearance. She has a 15-year history of osteoarthritis and is scheduled for hip replacement surgery in 2 days. She has been taking a variety of over-the-counter pain medications and has increased the dose and frequency in the past 6 months. Her vital signs are within normal limits. Physical examination shows no abnormalities. Laboratory studies show: Test Patient Serum creatinine 2.0 mg/dL (N: 0.6-1.2) Urine color Pale yellow Urine specific gravity 1.005 (N: 1.005-1.030) Urine microscopy Negative for casts and crystalsExamination of a renal biopsy specimen shows eosinophilic infiltration and diffuse parenchymal inflammation. Which of the following is the most likely cause of this patient's deterioration in renal function? A. Acute tubular necrosis B. Drug-induced interstitial nephritis C. IgA nephropathy D. Rapidly progressive glomerulonephritis E. Focal segmental glomerulosclerosis

B. Drug-induced interstitial nephritis

1. A 33-year-old woman comes to the physician because of a 2-day history of fever, chills, dyspnea and cough with purulent sputum. She says that she has cough and sore throat for the past six days for which she is taking over-the-counter cough suppressants. She has a 7-year history of bronchial asthma. Her temperature is 39.4°C (103ºF) and respirations are 24/min. Physical examination shows ill-looking woman with bronchial breath sounds in the left lower lung fields. A chest x-ray shows homogenous opacity in the left lower lobe. A sputum smear is positive for Gram-positive diplococci. Which of the following additional physical findings is most likely in this patient? A. Hyperresonant percussion note in the left lower lung B. Dull percussion note in the left lower lung fields C. Diffuse crackling sound in the entire left lung D. Deviation of the trachea to the right side E. Stony dull percussion note in the right lower chest

B. Dull percussion note in the left lower lung fields

A 32-year-old woman comes to the emergency department because of a 1-hour history of left-sided chest pain and dyspnea. She has a 6-year history of emphysema. She is in moderate distress and her respirations are 28/min. Physical examination shows centrally positioned trachea, hyper-resonant percussion note, and absent breath sounds in the upper left chest fields. Which of the following findings is most likely in his chest x-ray? A. Homogenous opacity in left lower lung field B. Hyperlucency & absent lung markings in the left upper chest field C. Wedge-shaped opacity in the left upper chest field D. Obliteration of the left costophrenic angle E. Hyperlucency of both lung fields

B. Hyperlucency & absent lung markings in the left upper chest field

A 71-year-old man comes to the physician for a follow up examination. He has a 15-year history of chronic bronchitis. Physical examination shows a loud P2 and parasternal heave. Right heart catheterization studies show elevated mean pulm pressure and decreased pulm wedge pressure. Pulm function studies show severe obstructive lung disease. Which of the following is most likely the major pathophysiologic mechanism that contributes to the pulmonary hypertension in this patient? A. Elevated right-sided cardiac output B. Hypoxic vasoconstriction C. Increased dead space D. Decreased cross-sectional area of pulm vascular bed E. Activation of renin-angiotensin-aldosterone system

B. Hypoxic vasoconstriction

A 69-year-old man comes to the physician because of 8-month history of difficulty in breathing. It was insidious in onset and progressive in nature. It is associated with a dry cough. Physical examination shows digital clubbing. Respiration is 24/min, and heart rate is 90/min. Pulmonary function tests show a restrictive pattern. Which of the following examination finding could support the diagnosis in this case? A. Soft P2 B. Inspiratory crackles C. Stony dull percussion note D. Hyperresonant breath sounds

B. Inspiratory crackles

12. A 55-year-old woman comes to the physician because of a 2-month history of fatigue, breathlessness, and difficulty balancing. She has a 3-year history of Grave's disease. Her temperature is 37°C (98.6°F), pulse is 90/min and respirations are 18/min. Physical examination shows conjunctival pallor, mild scleral icterus a wide gait with decreased sensation to fine touch over both feet. Deep tendon reflexes are sluggish. Laboratory studies show: Hemoglobin 7.8 g/dL (12-16g/dL) Mean corpuscular volume 112 fL (80-100fL) Total leukocyte count 3500/mm3 (4500-11,000/mm3). Which of the following additional findings is most likely in this patient? A. Increase in serum direct bilirubin B. Low reticulocyte count C. Low homocysteine levels D. Low methylmalonate levels E. Degeneration of Anterior column of spinal cord

B. Low reticulocyte count

6. A 30-year-old woman comes to the physician because of a 1-day history of fever, chills, and left flank pain. She does not take any medications or illicit drugs. Her menstrual cycles have been regular, and she is not currently sexually active. Her temperature is 39.5°C (103.1°F); other vital signs are within normal limits. Her abdomen is non-distended and soft with left costovertebral angle tenderness. Urine dipstick is positive for nitrite and leukocyte esterase, and urine microscopy shows pyuria. Which of the following histologic findings would most likely be associated with the patient's current condition? A. Interstitial fibrosis and lymphocytic infiltrate B. Neutrophils filling the lumen of the renal tubules C. Thickening of the capillaries and glomerular basement membrane D. Scarring of the glomeruli in a focal-segmental pattern E. Enlarged, hypercellular glomeruli with 'wire-looping' of capillaries

B. Neutrophils filling the lumen of the renal tubules

7. A 64-year-old man comes to the physician because of a 3-day history of left-sided chest pain and dyspnea. He was diagnosed with lung cancer 4-month ago and is on chemotherapy. Physical examination and imaging studies show a large left-sided pleural effusion. Thoracentesis is performed and the pleural fluid shows a milky white pleural fluid that fails to become clear upon centrifugation. Which of the following additional laboratory findings is most likely of this fluid? A. Elevated glucose level B. Pleural fluid Triglyceride to serum triglyceride ratio is >1.0 C. Pleural fluid Cholesterol to serum cholesterol ratio is >1.0 D. Absent Chylomicrons E. High amylase content

B. Pleural fluid Triglyceride to serum triglyceride ratio is >1.0

Patient 1 : Pulmonary function studies are performed and results show:FEV1/FVC ratio: 83% (N: >75%)Lung volumes and capacities: within normal limits,Diffusion capacity of lungs (DLCO): 20 (32 mL/min/mm Hg).Which of the following is the most likely cause of this patient's right heart failure? A. Systemic arterial hypertension B. Pulmonary arterial hypertension C. Coronary artery disease D. Mitral stenosis E. Diffuse interstitial lung disease F. Chronic obstructive lung disease

B. Pulmonary arterial hypertension

A 34-year-old woman is brought to the emergency department 25-minute history of severe chest discomfort and shortness of breath. She is recovering from an uncomplicated cholecystectomy performed 5 days ago. She has a 5-year history of systemic hypertension and dyslipidemia. She is taking oral contraceptive for the last 10 years. Vitals: Temp 37°C (98.6°F), Pulse 110/min, BP 116/74 and RR 28/min.Physical Examination: She is diaphoretic and hyperventilating.Cardiovascular, respiratory system examination shows no abnormalities.Laboratory: Blood gases: PaO2 65 mmHg (N:90-100) , PaCO2 32 mmHg (N: 35-40), pH 7.47 & HCO3 25 mEq/L. Which of the following is the most likely diagnosis? A. Pneumothorax B. Pulmonary embolism C. Pleural effusion D. Hypovolemic shock E. Septic shock

B. Pulmonary embolism

9. A 29-year-old man comes to the physician for a routine examination. He is an immigrant to USA from Kenya, Africa 8 months ago. He does not have any cough or fever. His vital signs are all within normal limits. Physical examination shows no abnormalities. An x-ray of his chest is normal. A PPD-skin test is performed which shows 12 mm (1.2 cm) induration. Which of the following is the next best step for this patient, according to CDC guidelines? A. Nothing to be done B. Send him to the nearest medical center for further evaluation to exclude active TB C. Repeat PPD-skin test after 2 weeks D. Vaccinate him with BCG vaccine E. Start with a single drug anti-TB treatment F. Start with multidrug anti-TB treatment

B. Send him to the nearest medical center for further evaluation to exclude active TB

7. A 55-year-old man is brought to the emergency department 30 minutes after he collapsed while getting up from the bed. He was recently started on thiazide diuretics. Physical examination shows dry mucus membranes and sunken eyes. His temperature is 37°C (98.6°F), pulse is 108/min, blood pressure is 88/50 mm Hg and respirations are 20/min. His urine output is 19 ml per hour. Laboratory studies show: BUN: 32 mg/dL (N: 7-18) Serum creatinine: 1.1 mg/dL (N: 0.6-1.2). Urine osmolality: 900 mosm/l (N 50-1200 mOsmol/kg H2O)Which of the additional laboratory findings are most likely in this patient? A. FeNa(fractional excretion of sodium) >2% B. Urine sodium <20mEq/L C. Urine Osmolality <350 mmol/L D. Urine/Plasma ratio of creatinine < 50 E. Urine Specific gravity 1.010

B. Urine sodium <20mEq/L -hypovolemic shock = prerenal kidney injury A is intrinsic renal failure

6. A 54-year-old woman comes to the physician because of a 5-month history of fatigue, shortness of breath and arthralgia. She was diagnosed with diffuse interstitial fibrosis 2 years ago. Physical examination shows loud P2 and right ventricular heave. An x-ray of the chest shows enlarged right cardiac border and dilated pulmonary trunk. Pressure studies show a mean pulmonary arterial pressure of 36 mmHg (Normal: 25). Which of the following best explains the pathogenesis of pulmonary hypertension in this patient? A. Elevated pulmonary capillary hydrostatic pressure B. Vascular destruction, inflammation and endothelial dysfunction C. Reduced endothelin production in pulm circulation D. Hypoxic vasoconstriction in the pulmonary vessels E. Formation of atherosclerotic plaques in the pulmonary artery

B. Vascular destruction, inflammation and endothelial dysfunction

A 67-year-old man with a 2-year history of pulmonary tuberculosis is diagnosed with a right-sided pleural effusion. He is prepared for thoracentesis. The needle for thoracentesis is inserted along the superior border of a rib to avoid damage to the neurovascular bundle that contains intercostal nerve, vein and artery. In cranio-caudal direction, which of the following correctly represents the locations of these structures? A. Nerve, artery, vein B. Vein, artery, nerve C. Artery, vein, nerve D. Nerve, vein, artery E. Artery, nerve, vein

B. Vein, artery, nerve

A 22-year-old woman comes to the physician because of an 8-month history of excess menstrual bleeding, easy bruising and recurrent nose bleeds. Her vitals are within normal limits and physical examination shows no abnormalities. Laboratory studies shows a normal PT, a slightly prolonged aPTT and a low plasma levels of both Factor VIII and vWF. Which of the following additional laboratory findings is most likely in this patient ? A. ↓ serum D- dimer B. ↓ platelet aggregation to ristocetin C. ↓ platelet aggregation to thrombin D. Giant platelets in the peripheral smear E. Platelet count less than 50,000/mm3

B. ↓ platelet aggregation to ristocetin

A 26-year-old woman is brought to the emergency department in unresponsive state because of a narcotic overdose. Her respirations are 8/min with shallow tidal volume. Physical examination shows altered consciousness, hypotension and cyanosis. Which of the following pulmonary function studies is the most appropriate next step in managing this patient? A. Estimating lung compliance B. Assessment of strength of the respiratory muscles C. Arterial blood gas studies D. Flow-volume studies E. Spirometry

C. Arterial blood gas studies

10. A 57-year-old man comes to the physician because of a 3-month history of a lump in his shoulder. He has been HIV positive for the past 10 years and has not taken his antiretroviral therapy. Physical examination shows enlarged, non-tender, supraclavicular lymph nodes. There is no hepatosplenomegaly. Laboratory studies show: Hemoglobin 9.9 g/dl (13.5-17.5. g/dl); Platelet count 190,000/mm3 (150,000-400,000/mm3); Total leukocyte count 9000/mm3 (4,500-11,000/mm3).Histologic examination of a lymph node biopsy shows replacement by a diffuse proliferation of a monomorphous population of lymphoid cells twice the size of normal lymphocytes, with high proliferative index. Which of the following is the most likely underlying cause of this patient's condition? A. PTEN gene mutation B. BCL-1 gene overexpression C. BCL-2 gene overexpression D. JAK-2 mutation E. CALR mutation

C. BCL-2 gene overexpression

6. A 24-year-old man comes to the physician for a pre-employment evaluation. His only complaint is that he develops bruises quite easily and frequently has nosebleeds. He denies any history of illness or regular medication use. Physical examination shows that vital signs are within normal limits. He has no petechiae, ecchymoses, or purpura. No lymphadenopathy or hepatosplenomegaly is evident. Laboratory tests show: WBC count: 11,400/mm3; Hemoglobin: 13.3 g/dL; Hematocrit: 39%; Platelet count 206,000/mm3; Partial thromboplastin time: Elevated ; Prothrombin time: Normal. Ristocetin-induced platelet aggregation: reduced. Which of the following molecular process is most likely impaired in this patient? A. Activation of factor IX B. Activation of prothrombin to thrombin C. Binding between collagen and platelet glycoproteins D. Formation of covalent bonds between fibrin monomers and chains E. Inactivation of factors V and VIII

C. Binding between collagen and platelet glycoproteins

11. An 83-year-old man is brought to the emergency department because of a 2-hour history of altered mental status. He has a 50 pack-year history of cigarette smoking. His respirations are normal. Arterial blood studies show pH=7.34, PaCO2=42 mm Hg, PaO2=92 mm Hg, HCO3=18 mEq/L, SaO2=72% and elevated blood lactate. Pulse oximeter shows an O2 saturation (SpO2) of 99%. Which of the following etiologies is the most likely cause of his altered mental status? A. Acute cerebral hemorrhage B. Acute respiratory failure C. Carbon monoxide poisoning D. Overdose of sleeping pill E. Chronic respiratory failure

C. Carbon monoxide poisoning

3. A 53-year-old woman comes to the physician because of a 3-day history of fever and right-sided flank pain. Her temperature is 38.4°C (101.1°F). Physical examination shows a non-distended, soft abdomen with right costovertebral angle tenderness. Laboratory tests show: Test Patient Urinalysis sp gr 1.010 (N: 1.005-1.030) pH 7.5 Blood 1+ Negative for glucose, protein, or ketones Urine microscopy Numerous WBCs and WBC castsAn x-ray of the abdomen shows a radiopaque calculus forming a cast of the right renal pelvis. Urine culture grows Proteus vulgaris. Which of the following crystals is most likely present on microscopy of this patient's urine? A. Sheafs of wheat B. Dumbbell-shaped C. Coffin-lid D. Square envelopes E. Needle-shaped

C. Coffin-lid

10. An 18-year-old woman is brought to the emergency because of a 1-month history of increasing fatigue, weight loss, urinary frequency, thirst, and hunger. She has a family history of Grave's disease in her mother and Addison's disease in her sister. She is suspected of having an autoimmune condition and advised treatment. She is counselled on the importance of treatment compliance to avoid potential life-threatening complications. Which of the following glomerular histology findings will be most likely observed on light microscopy if the patient were to develop renal complications secondary to her current condition? A. Endothelial and mesangial proliferation with neutrophils B. Normal appearance with or without mesangial proliferation C. Diffused and nodular mesangial expansion and hyaline thickening of both afferent and efferent arteriole D. Basement membrane duplication E. Membranous glomerulopathy with thick capillary walls

C. Diffused and nodular mesangial expansion and hyaline thickening of both afferent and efferent arteriole

A 34-year-old fire fighter is admitted to the hospital because of severe burns on his face and trunk. Physical examination shows second, and third degree burns and multiple fractures of his limbs. On the 3rd day, he develops severe dyspnea, tachycardia, tachypnea and hypotension. ABG studies show pH=7.50; PaCO2=26; PaO2=45 with SaO2 of 69%. PaO2/FiO2 ratio= 215; Pulm wedge Pr = Normal range. Chest x-ray shows bilateral, diffuse alveolar infiltrates. Which of the following pathogenic processes in the lungs is the most likely cause of this patient's dyspnea? A. Excess lung surfactant secretion B. Occlusion of distal pulm arteries C. Endothelial damage and capillary leak D. Overproduction of proteins by lung tissue E. Elevated pulm capillary hydrostatic pressure

C. Endothelial damage and capillary leak

A 23-year-old man comes to physician because of a 2-week history of protracted bleeding from cuts and scrapes. He has no family history of bleeding disorders. He was diagnosed to have Crohn's disease 2 years ago and had a surgical resection of ileum 6 months ago. Physical examination shows pallor. Laboratory studies shows prolongation of PT and aPTT but normal platelet counts. Deficiency of which of the following factor/s is/are the most likely cause of this patient's bleeding? A. Factor VIII B. Factor XIII C. Factor II, VII, IX, X D. Factor VIII, IX, XI E. Von Willebrand factor

C. Factor II, VII, IX, X

A 78-year-old patient is admitted and treated for pneumonia with IV antibiotics. The initial platelet count is 450,000 cells/microliter. The patient was initially debilitated and immobile but slowly improved with therapy. He was placed on prophylactic unfractionated heparin for DVT prevention. One-week later platelet count is 120,000cells / microliter. No further investigations were done. Three days later, the patient developed necrotic lesions over the abdomen. The following day left leg swelling is noted. Which of the following is the most likely diagnosis for this patient? A. Immune thrombocytopenic purpura B. Hemolytic uremic syndrome C. Heparin-induced thrombocytopenia D. Disseminated intravascular coagulation

C. Heparin-induced thrombocytopenia

An 18-year-old female comes to the primary care physician because of easy bruising for the past 2 weeks. She was diagnosed with a para-pharyngeal abscess a month ago and was treated with antibiotics for 2 weeks. CBC shows a platelet count of 14000/µL. PT and aPTT are normal. Physical examination shows (see next slide) Which of the following is the most likely diagnosis? A. Hemophilia A B. Glanzmann thrombasthenia C. Immune thrombocytopenic purpura D. Hemolytic uremic syndrome E. Disseminated intravascular coagulation

C. Immune thrombocytopenic purpura

11. A 53-year-old woman comes to the physician because of a 1-hour history of shortness of breath and left-sided chest pain. Her BMI is 33 kg/m2. She has a one-year history of deep vein thrombosis. Her pulse is 110/min, respirations are 28/min and temperature is 37.2°C. Physical examination shows swollen, tender right leg. Laboratory studies show elevated total white cell count and elevated serum D-dimer. A ventilation/perfusion (V/Q) scan shows perfusion defects of two lower segments of the left lung with normal ventilation. Which of the following changes is most likely in her lungs? A. Non-cardiogenic pulmonary edema B. Increase in diffusing capacity of lung C. Increase in total dead space D. Decrease in A-a gradient E. Increase in airway resistance

C. Increase in total dead space

A 64-year-old male is hospitalized with a transient ischemic attack and is evaluated for carotid disease. The physical exam is normal. CBC on admission is normal. The patient is started on heparin. A repeat CBC 1- one week later shows an Hgb of 14 g/dL (normal is 13 to 18 g/dL), WBC of 9,000/µL, and platelet count of 10,000/µL. The patient described develops thrombosis of the brachial artery. The next step in management would be to start the patient on A. Aspirin B. Steroids C. Lepirudin D. Increased Heparin E. Warfarin

C. Lepirudin

An 18-year-old man comes to the emergency department with a 1-hour history of nosebleeds. He has a history of recurrent gum bleeds when brushing his teeth. His mother has a history of menorrhagia. Laboratory tests reveal increased bleeding time and slightly increased partial thromboplastin time. The patient was diagnosed with Von Willebrand disease. In addition to the Von Willebrand factor (vWF), which of the following is also found in the alpha granules of the platelet? A. ADP B. Calcium C. Platelet factor 4 D. Serotonin E. Fibrin

C. Platelet factor 4

3. A 42-year-old man comes to the physician because of a 10-month history of daytime somnolence, increased fatigability, and morning headaches. He says that his sleep is disturbed at night, and he wakes up frequently with choking sensations. His wife complains that he snores loudly and occasionally stops breathing for a few seconds. His BMI is 32 kg/m2 (n:19-25). His pulse is 86/min and blood pressure is 152/92 mmHg. Physical examination shows large neck circumference. The soft palate, uvula, and pillars are not visible when he is asked to protrude his tongue. Which of the following complications is most likely in this patient? A. Renal failure B. Bronchiectasis C. Pulmonary hypertension D. Anemia E. Alcoholic fatty liver disease

C. Pulmonary hypertension

A 41-year-old man comes to the physician because of a 9-month history of progressive exercise intolerance. He does not have any pulmonary or other chronic diseases. Physical examination shows loud P2 in pulmonary area. Transthoracic echocardiography (TTE) shows right ventricular hypertrophy. V/Q scan shows no abnormalities. Which of the following mechanisms most likely increases the pulmonary vascular resistance (PVR) and thus contributes to pulmonary hypertension in this patient? A. Increase in cross-sectional area of the pulmonary bed B. Hyperventilation C. Pulmonary vascular remodeling D. Hypovolemia E. Activation of sympathetic nervous system

C. Pulmonary vascular remodeling

A 71-year-old man with a 10-year history of rheumatoid arthritis is diagnosed to have pulmonary hypertension. Physical examination shows parasternal heave and loud S2 in pulmonary area. Which of the following ECG findings is most likely suggestive of right ventricular hypertrophy in this patient? A. Decreased P wave amplitude B. Inconsistent R-R intervals C. R wave/S wave ratio greater than one in lead V1 D. Left axis deviation E. Left bundle branch block pattern

C. R wave/S wave ratio greater than one in lead V1

8. A 59-year-old man comes to the physician because of a 6-week history of dry cough and weight loss. He is a non-smoker. A chest x-ray shows a radiopaque mass in the apex of the left lung. Photomicrograph of a biopsy of this mass is shown in the figure. Which of the following complications is most likely in him? A. Cushing's syndrome B. SIADH C. Recurrent thrombophlebitis D. Renal stones E. Episodes of flushing

C. Recurrent thrombophlebitis

5. A 23-year-old man comes to the physician because of a 10-month history of progressive shortness of breath and yellow eyes. He is a non-smoker and does not consume alcohol. His vital signs are all within normal limits. Physical examination shows hyperresonant percussion note and diminished breath sounds over lower lung fields bilaterally. A chest X-ray shows hyperlucency in the lower lobes. Laboratory studies show elevated levels of hepatic transaminases and bilirubin. Which of the following diagnostic tests is most likely to help in establishing the etiology of this patient's condition? A. Spirometry for FEV1/FVC B. Lung biopsy C. Serum alpha-1 antitrypsin D. High resolution CT scan E. Serology for autoantibodies

C. Serum alpha-1 antitrypsin

5. A 3-year-old boy is brought to the pediatrician by his mother because of a 1-month history of irritability and decreasing appetite. He was previously well except for surgical repair of hypospadias at 1 year old. His blood pressure is 120/80 mmHg. Physical examination shows an enlarged abdomen with a palpable mass in the right lumbar region that does not cross the midline. A CT scan of the abdomen shows a 10-cm solid mass in the right kidney. Surgery is planned for excision and biopsy of the mass. Which of the following histologic findings is most likely expected in this patient's mass? A. Clear cells with cytoplasm rich in lipids and glycogen B. Tubulointerstitial fibrosis with presence of corticomedullary cysts C. Sheets of small round blue cells with primitive stromal and tubular structures D. Dyscohesive papillary structures with central fibrovascular cores showing loss of polarity E. Radially arranged elongated cysts with tubular dilation

C. Sheets of small round blue cells with primitive stromal and tubular structures

9. A 39-year-old man comes to the physician because of a 6-month history of intermittent cough, dyspnea and wheeze. He was diagnosed with cirrhosis of the liver 2 years ago. There is no family history of asthma. He smokes one packet of cigarettes per day for the past ten years. Physical examination shows tachypnea, retraction of scalene and intercostal muscles, hyperresonant percussion note and decreased breath sound in all lung fields. Laboratory studies show a decreased serum apha-1 antitrypsin and a reduced diffusing capacity of lungs. Which of the following complications is most likely in this patient? A. Pleural effusion B. Spontaneous primary pneumothorax C. Spontaneous secondary pneumothorax D. Lung abscess E. Atelectasis

C. Spontaneous secondary pneumothorax

A 26-year-old, previously healthy white man comes to the physician because of a 6-week history of early satiety. He also has a 3-day history of abdominal pain and constipation. Physical examination shows a non-tender mass in the right lower quadrant of abdomen. CT scan of the abdomen as shown. Immunophenotyping of the mass shows CD10, CD19 and CD20 and; cytogenetic analysis shows t(8,14). Histopathological examination of the mass is likely to show which of the following features? A. Large size, irregular nuclei and prominent nucleoli. B. Mixed small and large cells arranged in follicles C. Uniform cells with many mitoses, apoptotic cells, and tingible body macrophages. D. Uniform cells with cyclin D1 nuclear staining E. Immature cells with expression of CD3 and TdT

C. Uniform cells with many mitoses, apoptotic cells, and tingible body macrophages.

12. A 21-year-old man comes to the physician because of a 2-day history of blood in his urine, cough and hemoptysis. His temperature is 98.4°F(37°C), and blood pressure is 170/116 mm Hg. Physical examination shows no abnormalities. Laboratory studies show a blood urea nitrogen of 39 mg/dL (8-21 mg/dL) and creatinine level of 4.1 mg/dL (0.6-1.2 mg/dL). A renal biopsy specimen is obtained; the immunofluorescence pattern of staining with antibodies against human IgG is shown. Which of the following is the most likely cause of renal injury in this patient? A. C3 convertase antibodies B. Monoclonal IgG antibodies C. Antistreptococcal antibodies D. Anti-glomerular basement membrane antibodies E. Anti-double stranded DNA antibodies

D. Anti-glomerular basement membrane antibodies

2. A 3-year-old boy is brought to the clinic because of a 1-month history of polyuria and polydipsia. He has no history of vomiting, sore throat, joint pain, head trauma, or medication use. Physical examination shows weight and height are below the 25th percentile. He has mild dehydration with a low-normal blood pressure. Laboratory tests show shows hypocalcemia, hypokalemia, metabolic alkalosis, and high serum levels of renin and aldosterone. Urine electrolytes shows a high urine chloride. Which of the following is the most likely diagnosis? A. Primary hyperaldosteronism B. Liddle syndrome C. Gitelman syndrome D. Bartter syndrome E. Congenital adrenal hyperplasia

D. Bartter syndrome

A 65-year-old man comes to the physician because of a 1-month history of easy fatiguability and widespread ecchymosis. His temperature is 37.2°C (99°F) and blood pressure is 130/80mm Hg. Physical examination shows enlargement of liver and spleen. Stool sample is positive for occult blood. Laboratory tests show hemoglobin 9.0g/dl (13.5-17.5 g/dl), platelets 16,000/mm3 (150,000-400,000/mm3),and WBCs 80,000 (4,500-11,000/mm3). Plasma D-dimer, fibrin degradation products and prothrombin time is high. A diagnosis of AML is made, and a bone marrow biopsy is done for cytogenetic analysis shows t(15;17). Which is the most likely immunophenotypic finding ? A. CD15, CD30 B. CD11, CD14 C. CD41, CD42,CD61 D. CD13,CD33 E. CD19,CD20,CD23

D. CD13,CD33

4. A 65-year-old man comes to the physician because of a 2-year history of progressive shortness of breath. He says his problem worsens on routine exercise like evening walk. There was no history of rash, joint pains, chest pain or heart disease. Physical examination shows a diffuse bilateral fine inspiratory crackle. Laboratory studies show increased erythrocyte sedimentation rate, negative RA factor and antinuclear antibody test. A high-resolution CT scan shows ground glass opacities, traction bronchiectasis and honeycombing in the peripheral regions of lower thorax bilaterally. Which of the following pulmonary function study findings is most likely in this patient? FEV/FVC,FRC,TLC,DLCO A. A high high low high B. B high high high low C. C low low low low D. D normal low low low E. E normal low high high

D. D normal low low low

A 54-year-old woman comes to the physician because of a 6-month history of progressive shortness of breath. She was diagnosed with chronic thromboembolic disease 1 year ago. Diagnostic studies show elevated mean pulmonary arterial pressure, decreased pulm wedge pressure and abnormal V/Q. Which of the following is most likely the major pathogenic mechanism to cause pulmonary hypertension in this patient? A. Increase in cardiac output of the right ventricle B. Hypoxic vasoconstriction C. Increased left atrial pressure D. Decreased cross-sectional area of pulm vascular bed E. Activation of renin-angiotensin-aldosterone system

D. Decreased cross-sectional area of pulm vascular bed

A 30-year-old male comes to the emergency with complaints of decreased urination and confusion for 2 days. He has had several episodes of epistaxis and gum bleeding since the illness began. His past medical history is unremarkable, and the patient is not on any medications. Vitals are Temperature 39C, blood pressure is 100/62 mm Hg, the pulse is 95/min, and respirations are 18/min. Physical examination shows a purpuric rash, bilaterally in the lower extremities. The peripheral blood smear is shown in the picture. Which of the following is the most likely mechanism behind the patient's illness? A. Decreased levels of von Willebrand Factor (vWF) B. Defect in platelet aggregation C. Peripheral platelet destruction D. Deficiency in degradation of von Willebrand Factor (vWF) multimers E. Defect in platelet-to-collagen adhesion

D. Deficiency in degradation of von Willebrand Factor (vWF) multimers

A 33-year-old female comes to the ER because of the sudden onset of dyspnea. She also has associated symptoms of feeling tired, exhausted on minimum excursion, and difficulty swallowing food for the past six months. Upon physical examination, she has ptosis of both eyelids. On performing the pulmonary function tests, which of the following parameters is most likely to remain normal in this case? A. Total lung capacity B. Forced expiratory volume C. Residual volume D. Diffusing capacity of lung

D. Diffusing capacity of lung

9. A 20-year-old man comes to the physician because of a 5-day history of fatigue and dark-colored urine. He also has a 7-day history of pharyngitis. He has had similar episodes in the past but never received a diagnosis. He is not aware of any similar history in his family members. His temperature is 37.7°C (100°F), pulse is 90/min and respirations are 18/min. Physical examination shows conjunctival pallor and icterus. Laboratory studies show: Hemoglobin 7.8 g/dL (12-16g/dL) Total leukocyte count 6300/mm3 (4500-11000/mm3) Platelet count 160000/mm3 (150000-400000/mm3) Haptoglobin 15 mg/dl (50-220 mg/dl)Peripheral blood smear is shown in photomicrograph. Also shown is supravital stain showing dark spherical inclusions in the RBCs.He is suspected to have a deficiency of an enzyme in his RBCs.Which of the following substrates does the most likely deficient enzyme use? A. 6-phosphogluconate B. Ribose-5-phosphate C. Glutathione D. Glucose-6-phosphate E. NADPH

D. Glucose-6-phosphate

A 67-year-old man presents to the emergency room with complaints of abdominal distention and easy bruising. The patient consumes half a bottle of rum a day in order to function and takes no medications currently. Physical examination shows multiple ecchymoses all over the body, scratch marks, gynecomastia, ascites, and a palpable spleen 3 cm below the left costal margin. Which of the following is the most likely mechanism of the ecchymosis in this patient? A. Immune-mediated destruction B. Thrombotic thrombocytopenic purpura C. Sepsis D. Hypersplenism E. Aplastic crisis

D. Hypersplenism

A 61-year-old man comes to the physician because of a 4-day history of fever, productive cough, right-sided chest pain and dyspnea. He has a 20 pack-year of cigarette smoking. His temperature is 39°C (102°F) and respirations are 20/min. Physical examination shows fullness and diminished movement of the chest wall, stony dull percussion note, and diminished breath sounds in the lower right lung fields. An x-ray of his chest shows homogenous opacity in the right lower lung area and blunting of the right costophrenic angle. Sputum studies show a Gram-positive, alpha-hemolytic, lancet-shaped diplococci. Which of the following pathophysiologic mechanisms is the most likely cause of pleural effusion in this patient? A. Decreased colloid oncotic pressure of plasma B. Increased blood volume in systemic circulation C. Blockage of the thoracic lymph vessels D. Inflammation of the pleura secondary to the lung infection E. Elevated pulmonary capillary hydrostatic pressure

D. Inflammation of the pleura secondary to the lung infection

1. A 66-year-old man is brought to the emergency department because of a 15-min history of left-sided chest pain and dyspnea. He has a 20-year history of chronic obstructive pulmonary disease and smoked 1½ packs of cigarettes daily for 25 years. His respirations are 26/min and rest of the vital signs are within normal limits. Physical examination shows centra trachea, absent tactile fremitus, hyper-resonant percussion notes and absent breath sounds in the left upper lung fields. A decrease in which of the following pulmonary factors is most likely in the affected lung of this patient? A. Airway resistance B. Lung perfusion C. Alveolar dead space D. Lung compliance E. Intrapleural pressure

D. Lung compliance

4. A 39-year-old man comes to the physician because of a 2-week history of fever, shortness of breath and cough with scanty sputum. He was diagnosed with HIV infection 2 years ago and is noncompliant with anti-retroviral treatment. His temperature is 38.5°C (101°F) and respirations are 28/min with shallow tidal volume. Physical examination shows crackles all over the chest. A chest x-ray is shown in the figure. Laboratory studies show a CD4+ T cell count of 165/mm3 (N: >500). A bronchoalveolar lavage is performed for further study. Which of the following laboratory techniques is most likely to confirm the etiologic agent? A. Ziehl-Neelsen staining of the specimen for Acid-fast bacteria B. Periodic acid-Schiff staining of the specimen for glycogen C. India ink staining of the specimen for yeast D. Methenamine sliver staining of the specimen for yeast E. Giemsa staining of specimen for inclusion bodies

D. Methenamine sliver staining of the specimen for yeast

A 53-year-old woman comes to the physician for a follow up examination. She had an artificial heart valve 3 months ago and she is currently on a low dose oral warfarin. She is symptom free, and her vital signs are within normal limits. Which of the following combination of laboratory results is most likely in this patient? BT/Platelet Count/PT/aPTT A. ↑ ↑ N N B. N N ↑ ↑ C. ↑ N ↑ N D. N N ↑ N E. N ↑ N N

D. N N ↑ N

A 29-year-old man comes to the local health clinic for tuberculosis screening. He is an immigrant to USA from New Zealand, 2 years ago. He does not have any symptoms of tuberculosis. Physical examination shows no abnormalities. A PPD skin test is performed, and a 12 mm induration is observed. His chest x-ray is normal. Which of the following is the most appropriate interpretation of the PPD skin test and suitable management for this patient? A. PPD is positive but no anti-TB therapy indicated B. PPD is positive and INH prophylaxis indicated C. PPD is considered as borderline positive D. PPD is negative and no treatment required E. PPD is negative but he receives INH prophylaxis

D. PPD is negative and no treatment required

A 60-year-old man comes to the physician because of 2-month history of constant, dull pain in his ribs and back. He also has a 1-week history of headache, nausea and increased urination. Physical examination shows tenderness in the ribs and spine. Laboratory studies show: A. Bacteria B. White cell casts C. Lymphoplasmacytic cells D. Proteins E. Red cell casts

D. Proteins

6. A 68-year-old woman with known COPD presents with shortness of breath and cough for the past 2 weeks. Respirations are 23/min and oxygen saturation is 93% on room air. On physical examination, the patient appears uncomfortable and bilateral wheezes are heard. Arterial blood gas analysis shows a pH of 7.23, PCO2 of 77 mm Hg, and PO2 of 56 mm Hg.Which of the following mechanisms best explains this patient's arterial blood gas values? A. Decreased pulmonary capillary blood flow B. Decreased total lung volume C. Increased hemoglobin oxygen binding capacity D. Increased tidal volumes E. Increased alveolar dead space

E. Increased alveolar dead space

5.A 65-year-old man comes to the physician because of 1-month history of widespread ecchymoses on his skin. He was diagnosed with mucinous adenocarcinoma of the rectum 6 months ago. Physical examination shows cachexia and pallor. An abdominal CT scan shows multiple hepatic masses. Laboratory studies show PT 30 sec, (11-15), aPTT 55 sec (25-40), platelet count 15,200/mm3 , fibrinogen level 75 mg/dL (normal 150 -300 mg/dL), and elevated fibrin split product levels (D-dimer). Which of the following morphologic findings is most likely present on examination of his peripheral blood smear? A. Howell-Jolly bodies B. Teardrop cells C. Macro-ovalocytes D. Schistocytes E. Target cells

D. Schistocytes

A 32-year-old woman comes to the office for a routine health maintenance examination. The patient's last visit to the office was 1 year ago, and today she says she is "doing about the same", except for fatigue and an unintentional 5-kg (11-lb) weight loss and fatigue. She has a history of renal transplantation at age 15 years performed for end-stage renal disease as a result of post-streptococcal glomerulonephritis. Physical examination shows two enlarged, matted, nontender lymph nodes in the left anterior cervical chain and a firm, nontender 1.5 cm lymph node in the right groin. A diagnosis of lymphoma is suspected, and biopsy of the cervical lymph node is sent for histopathology and immunophenotyping. Which of the following statements best explains the lymphoid malignancies? A. All B-cell lymphomas are aggressive B. All T-cell lymphomas are aggressive C. All lymphomas do not necessarily contain clonal lymphoid cells D. The distinction between Hodgkin and non-Hodgkin lymphomas is important because the treatments and prognosis are distinct E. T-cell lymphomas are more common in the Western countries than B-cell lymphomas

D. The distinction between Hodgkin and non-Hodgkin lymphomas is important because the treatments and prognosis are distinct

11. A 35-year-old man comes to the physician because of a 4-week history of abdominal distension and swelling of his legs and feet. He has a 10-year history of hepatitis B and a 2-year history of hypertension. Current medications are metoprolol and amlodipine. His blood pressure is 130/80 mm Hg, pulse is 80/min and respirations are 16/min. Physical examination shows bilateral pitting pedal edema. Laboratory studies show deranged liver function tests.Urinalysis shows proteinuria (5.6 g/day) without hematuria or RBC casts. Which of the following light microscopic findings is most likely in a renal biopsy of this patient? A. Normal appearing glomeruli B. Focally sclerosed glomeruli C. Mesangial and endothelial proliferation with neutrophils D. Thickened basement membrane E. Fibrotic crescents

D. Thickened basement membrane

A 28-year-old healthy man volunteers for a pulmonary study. He is made to sit in an erect position during the study. His ventilation (V), perfusion (Q) and V/Q ratio in upper, middle and lower parts of the lungs are measured. Which of the following regarding ventilation and perfusion of the lungs is most likely? A. Maximum ventilation is in the upper lungs B. Maximum perfusion is in the upper lungs C. Least perfusion in the middle lungs D. V/Q ratio is maximum in the upper lungs E. V/Q ratio is same in all parts of the lungs

D. V/Q ratio is maximum in the upper lungs

10. A 56-year-old man comes to the physician for a follow up examination. He has a 16-year history of COPD and a 15 pack-year history of smoking. He is advised to undergo pulmonary function studies. The report, using spirometry and helium dilution technique shows:Functional residual capacity = 3.5 L Inspiratory reserve volume = 2.5 L Residual volume = 2.0 L Vital capacity = 4.5 LWhich of the following is most likely the Total lung capacity? A. 3.8 L B. 4.5 L C. 4.8 L D. 5.5 L E. 6.5 L F. 7.5 L G. 8.5 L

E. 6.5 L TLC = VC+RV

8. A 62-year-old man is brought to the emergency dept. because of a 1-week history of dyspnea, cough and altered mental state. He has a 20-year history of chronic obstructive lung disease. His pulse is 98/min and respirations are 26/min. His PaO2 is 66 mmHg and O2 saturation at room air is 81%. Which of the following ABG values is most likely in him? (Normal:PaCO2=40; HCO3=24; pH=7.4) PaCO2 [HCO3] pH A. 50 36 7.13 B. 25 30 7.71 C. 40 25 7.40 D. 30 21 7.31 E. 60 30 7.33

E. 60 30 7.33

3. A 51-year-old man comes to the emergency department because of a 1-day history of severe dyspnea with exertion. He says that the dyspnea worsens while walking short distances. He has a 10-year history of bronchial asthma and uses inhaler. He has smoked 1 pack of cigarettes daily for 20 years. His pulse is 92/min, respirations are 22/min with increased tidal volume and blood pressure is 126/78 mm Hg. Physical examination shows diffuse rhonchi all over the chest. Arterial blood gas studies show a PaO2 of 68 mm Hg and PaCO2 of 26 mm Hg. Which of the following is the most likely cause of this patient's blood gas findings? A. Decreased chest wall compliance B. Increased dead space C. Poor respiratory drive D. Respiratory muscle fatigue E. Alveolar hyperventilation

E. Alveolar hyperventilation

A 65-year-old man comes to the physician because of a 10-month history of gradual weight loss, weakness, anorexia, and easy fatigability. Physical examination shows marked splenomegaly. CBC shows: Hemoglobin, 11.9 g/dl (13.5-17.5. g/dl); Hematocrit, 39.1% (41%-53%); MCV, 92 µm3 (80-100 µm3); Platelet count, 589,000/mm3 (150,000-400,000/mm3); and WBCs, 145,000/mm3 (4,500-11,000/mm3). PBF shows a marked leukocytosis with neutrophilia, a few myelocytes and promyelocytes, rare blasts, and a basophilia (4300/μl). The BM aspiration and biopsy reveal a markedly hypercellular BM and a markedly increased myeloid : erythroid ratio of 29:1. Dysregulation of which of the following genes is most likely to be involved in the pathogenesis of this disease? A. BCL6 B. MYC C. CCND1 (gene encoding cyclin D1) D. JAK2 E. BCR-ABL

E. BCR-ABL

1. A 33-year-old woman comes to the physician because of a 1-week history of progressive hematuria and flank discomfort. She has a 3-year history of recurrent urinary tract infections adequately treated with antibiotics. Her temperature is 101.3°F (38.5°C). Physical examination shows conjunctival pallor and right costovertebral angle tenderness. Intravenous urography is shown on the right. Which part of the kidney is the most likely origin of this patient's condition? A. Proximal tubule B. Descending limb of loop of Henle C. Ascending limb of loop of Henle D. Distal convoluted tubule E. Collecting duct

E. Collecting duct

2. A 65-year-old man comes to the physician because of a 7-month history of worsening shortness of breath and a mild non-productive cough. He has no history of fever, chills or hemoptysis. He has smoked 1 pack of cigarettes daily for 45 years. His respiratory rate is 28/min and rest of his vital signs are within normal limits. Physical examination shows a thin man in respiratory distress with increased anteroposterior chest diameter, hyperresonant percussion note and diminished breath sounds in bilateral lung fields and loud S2 in pulmonary area. A chest X-ray shows hyperinflation of bilateral lung fields with flattening of the diaphragm. Pulmonary function studies show increased TLC and FRC but decreased FEV1/FVC. Which of the following additional diagnostic test findings is most likely in this patient? A. Increased vital capacity B. Decreased residual volume C. Increased mid-expiratory flow rate D. Increased absolute eosinophil count E. Decreased diffusing capacity of lung

E. Decreased diffusing capacity of lung

11. A 45-year-old man comes to the physician because of a 4-day history of fatigue, fever, and bleeding from gums. His temperature is 37.7°C (100°F), pulse is 110/min and respirations are 20/min. Physical examination shows conjunctival pallor, multiple petechiae, and purpura on the lower extremities. Laboratory studies show: Hemoglobin 8.8 g/dL (12-16g/dL) Total leukocyte count 5300/mm3 (4500-11000/mm3) Platelet count 4500/mm3 (150000-400000/mm3) Serum creatinine 1.5mg/dl (0.6-1.2mg/dl)A photomicrograph of peripheral blood smear is shown.Which of the following additional laboratory findings is most likely in this patient? A. Abnormal platelet aggregation tests B. Increased ADAMTS13 C. Decreased serum LDH levels D. Decreased serum iron E. Decreased serum haptoglobin

E. Decreased serum haptoglobin

5. A 48-year-old man is brought to the emergency department because of a 6-week history of fever, purulent sputum, altered mental state and dyspnea. He is a chronic alcoholic and was binge drinking 6 weeks ago. His temperature is 38.4°C (101°F) and respirations are 20/min. Physical examination shows increased vocal fremitus and amphoric breath sounds in the right lung base. An x-ray of the chest shows a 7-cm diameter, thick-walled cavity with air-fluid level in the right lower zone. Which of the following is the most likely diagnosis? A. Parapneumonic effusion B. Community acquired lobar pneumonia C. Bronchiectasis D. Nosocomial pneumonia E. Lung abscess

E. Lung abscess

A 41-year-old man comes to the physician because of a 5-day history of fever, malaise, headache, cough and dyspnea. Physical exam shows rales in right lung fields. Laboratory studies show positive Coomb's test and cold agglutinins. An x-ray of the chest shows patchy reticular consolidations in the right lower lobe. Which of the following pathogens is the most likely aetiologic agent? A. Staph. aureus B. Hemophilus influenzae C. Legionella pneumophila D. Strept. pneumoniae E. Mycoplasma pneumoniae

E. Mycoplasma pneumoniae

Which of the following most likely represent the relationship between pulmonary vascular resistance (PVR), Pulmonary artery pressure, Left atrial pressure and Cardiac output (CO)? A. PVR = CO x Pulm artery pressure B. PVR = CO/ Pulm artery pressure C. PVR = Pulm artery pressure/Left atrial pressure D. PVR = CO x Left atrial Pr E. PVR = (Pulm art Pr - Left atrial Pr)/CO

E. PVR = (Pulm art Pr - Left atrial Pr)/CO

A 65-year-old woman comes to the physician because of a 4-day history of fever, productive cough and dyspnea. Her sputum culture is positive for encapsulated Gram-positive cocci that are in pairs. An x-ray of the chest shows lobar pneumonia in the left lower lobe. The patient's dyspnea is primarily due to which of the following? A. Inadequate perfusion B. Increased ventilation C. Decreased airway resistance D. Increased lung compliance E. Poor oxygen diffusion

E. Poor oxygen diffusion

9. A 64-year-old man comes to the physician because of a 3-day history of swelling and puffiness around the eyes. He also has a 5-day history of pain and swelling in his right leg and a 20-year history of Diabetes. Current medications are Metformin, Ibuprofen and Dicloxacillin. His temperature is 37.4°C (99.4° F), pulse is 80/min, and blood pressure is 150/90mm Hg. Physical examination shows periorbital swelling; and tenderness, warmth and swelling in right leg. Laboratory studies show: Serum Urea nitrogen 38 mg/dL (N 8-21 mg/dL) Creatinine 3.0 mg/dL (N 0.6-1.2 mg/dL) Complement C3 levels 0.20g/L (N 0.88 to 2.01 g/L)Blood culture shows Staphylococcus aureus. Urinalysis shows RBCs, RBC casts and 2+ proteins. Which of the following is the most likely diagnosis? A. Drug induced Acute interstitial nephritis B. Acute pyelonephritis C. Membranous nephropathy D. IgA Nephropathy E. Post infectious glomerulonephritis

E. Post infectious glomerulonephritis

8. A 20-year-old man is brought to the emergency department 1 hour after a gunshot injury to his abdomen. His temperature is 36°C (96.8°F), pulse is 140/min, respirations are 24/min, and blood pressure is 90/60 mm Hg. During an exploratory laparotomy, a large laceration is found that includes the right and left lobes of the liver. Active bleeding is present. He is resuscitated with intravenous fluid and blood transfusion. 24 hours after surgery, the nurse observes that the urinary catheter has drained 50mls of urine despite adequate hydration. Results of laboratory studies are shown: Serum Urea nitrogen 55 mg/dL (N 8-21 mg/dL) Creatinine 3.9 mg/dL (N 0.6-1.2 mg/dL)Urinalysis shows muddy brown casts. These findings are suggestive of ischemia in which of the following renal structures? A. Interstitium B. Renal artery C. Glomerulus D. Collecting duct E. Proximal tubule

E. Proximal tubule

A clinical study is performed involving adult patients diagnosed with microangiopathic hemolytic anemia. A subgroup of patients who had fever or diarrhea preceding the initial diagnosis of anemia was included. The patients had schistocytes present on peripheral blood smears. Some of these patients were found to have a deficiency of a metalloproteinase known as ADAMTS13. Which of the following conditions would exhibit this deficiency? A. Disseminated intravascular coagulation (DIC) B. Hemolytic-uremic syndrome (HUS) C. Heparin-induced thrombocytopenia (HIT) D. Idiopathic thrombocytopenic purpura (ITP) E. Thrombotic thrombocytopenic purpura (TTP)

E. Thrombotic thrombocytopenic purpura (TTP)

4. A 72-year-old man comes to the physician because of a 1-year history of intermittent urinary hesitancy and dribbling. His symptoms have become worse in the past 6 weeks, and he wakes up 3 to 4 times a night to urinate. His vital signs are within normal limits. Physical examination shows no abnormalities. Digital rectal examination shows an enlarged prostate with a rubbery consistency. Stool studies show no abnormalities. An intravenous pyelogram is shown on the right. Which of the following is most likely responsible for this patient's imaging findings? A. Multiple endocrine neoplasia type 1 B. Autosomal dominant polycystic kidney disease C. Prerenal azotemia D. Renal cell carcinoma E. Urethral obstruction

E. Urethral obstruction -hydronephrosis obstruction after the bladder

A 48-year-old woman was evaluated and admitted to the medical floor following a new onset seizure activity. She was placed on bed rest. Five days later, she died after having sudden onset shortness of breath and tachycardia for fifteen minutes. The postmortem revealed she had a large pulmonary embolus. Which of the following is NOT a risk factor for pulmonary embolism? A. Lung cancer. B. Deep vein Thrombosis C. Use of oral contraceptive pills. D. Recent transatlantic flight of greater than 8 hrs. E. Utilizing compression stockings

E. Utilizing compression stockings

A 65-year-old man comes to the physician because of a 1-month history of easy fatiguability and widespread ecchymosis. His temperature is 37.2°C (99°F) and blood pressure is 130/80mm Hg. Physical examination shows enlargement of liver and spleen. Stool sample is positive for occult blood. Laboratory tests show hemoglobin 9.0g/dl (13.5-17.5 g/dl), platelets 16,000/mm3 (150,000-400,000/mm3),and WBCs 80,000 (4,500-11,000/mm3). Plasma D-dimer, fibrin degradation products and prothrombin time is high. A diagnosis of AML is made, and a bone marrow biopsy is done for cytogenetic analysis. What is the most likely karyotypic abnormality ? A. t(8;14) B. t(8;21) C. t(9;22) D. t(14;18) E. t(15;17)

E. t(15;17)

4. A 32-year-old female comes to the primary care physician because of easy bruising in her extremities for the past 2 weeks. She had a runny nose and cough a few weeks ago and was diagnosed with a viral illness and was treated with over-the-counter medications. She does not take any medications or use illicit drugs. Lab investigations shows a platelet count of 14000/µL. Physical examination shows Petechiae and purpura on the lower extremities. What additional lab findings is expected in this patient? A. ↑ D-dimers, schistocytes, ↓fibrinogen B. ↑ bleeding time, ↑ aPTT C. ↓ Hb, ↑ LDH, ↑ Cr (creatinine) D. ↑ aPTT, normal PT, and bleeding time E. ↑ anti-platelet factor 4 [PF4]/H antibodies F. ↑ Bleeding time

F. ↑ Bleeding time

A 15-year-old boy is brought to the emergency department because of a persistent nosebleed that has lasted for 30 minutes after a sporting incident. He had a runny nose and cough 6 weeks ago and has noticed his extremities bruising more easily over the past month. He does not take any medications or use illicit drugs. Physical examination reveals small non-blanching lesions on the lower legs. Laboratory studies show a platelet count of 35,000/mm3. Which of the following is the most appropriate next step in the evaluation of this patient? a. Repeat the complete blood count (CBC) b. Review the peripheral blood smear c. Obtain prior platelet counts if available d. Assess for other hematologic abnormalities

a. Repeat the complete blood count (CBC)

3. An 11-year-old boy is brought to the physician by his mother for evaluation of easy bruising. At the age of 10, he experienced hemorrhaging around the pharynx that produced acute airway obstruction. His male relatives have similar bleeding problems. Physical examination shows a marked reduction in joint mobility of the ankles, knees, and elbows. Laboratory studies show hemoglobin of 13.1g/dL and hematocrit of 39.2%. Which of the following is involved in the pathophysiology of this patient's condition? a. Autoantibodies directed against platelet membrane glycoproteins (e.g. GPIIb/IIIa) b. Deficiency of a factor of coagulation c. Production of inhibitory autoantibodies against ADAMTS13 d. Production of Shiga toxins that damage podocytes and tubular cells, and vascular endothelial cells e. Abnormal activation of coagulation and fibrinolysis

b. Deficiency of a factor of coagulation

1. A 40-year-old woman comes to the emergency room because of the sudden onset of a fever of 38.9oC, dark urine, and petechiae. Thrombotic thrombocytopenic purpura is suspected. Laboratory evaluation shows deteriorating renal function with a creatinine 3 times greater than her baseline. What additional laboratory finding is expected in this patient? Bleeding Time/aPTT/PT/Platelet Count a. 1 normal high normal normal b. 2 high high high low c. 3 normal normal high normal d. 4 high normal normal low e. 5 all normal

d. 4 high normal normal low

2. A 40-year-old woman presents to the emergency room because of purplish discoloration of the skin. Over 6 days ago had fatigue, malaise, diarrhea, and vomiting. Physical examination shows slight confusion, petechiae, and purpura on her lower extremities. Laboratory studies show a hematocrit of 25% and a platelet count of 26,000 cells/mm³. Lactate dehydrogenase is elevated. Serum creatinine is 2.1 mg/dL. Peripheral smear shows fragmented red blood cells and an elevated reticulocyte count. The urine dipstick shows hematuria and proteinuria. Which of the following is the most likely diagnosis of this patient? a. Immune thrombocytopenic purpura b. Heparin-induced thrombocytopenia c. Thrombotic thrombocytopenic purpura d. Hemolytic Uremic syndrome e. Disseminated intravascular coagulation

d. Hemolytic Uremic syndrome

7. A 69-year-old man is admitted to the hospital for management of a pulmonary embolus for which he receives heparin. Five days later, he reports bilateral leg swelling and pain. Ultrasonography reveals bilateral occlusive thrombi in the popliteal veins. His prothrombin time is 12 sec, and activated partial thromboplastin time is 65 sec. His heparin is discontinued. Which of the following suggest heparin-induced thrombocytopenia? a. New onset of thrombocytopenia (ie, platelet count <150,000/microL) b. A decrease in platelet count by 50 percent or more from baseline c. Arterial thrombosis d. Venous thrombosis e. Necrotic skin lesions at heparin injection sites f. All of the above

f. All of the above


Related study sets

Chapter 7: Identifying Advanced Attacks

View Set

Reading PPV (Purpose and Point of View)

View Set

chapter 12: Cardiovascular system- Blood

View Set